Location via proxy:   [ UP ]  
[Report a bug]   [Manage cookies]                

D Block & Co Ordination Compounds 33p

Download as pdf or txt
Download as pdf or txt
You are on page 1of 48

JEE

JEEADVANCED - VOL
MAINS - CW - VOL- VII
-I d-BLOCK & CO-ORDINATION COMPONDS
RELATIONS

d-BLOCK & CO-ORDINATION COMPOUNDS


SYNOPSIS Atoms of the transition elements are smaller than
those of the Group 1 or 2 elements in the same
d-BLOCK horizontal period.
1. Zn, Cd, Hg of group 12 (II B group) are not This is partly because of the usual contraction in
regarded as transition elements size across a horizontal period discussed above,
and partly because the orbital electrons are added
2. Elements with d 5 configuration:
to the penultimate d shell rather than to the outer
Cr :  Ar  4s1 3d 5 ; Mn :  Ar  4s 2 3d 5 shell of the atom.
8. Elements with similar size : Fe, Co, Ni
Mo :  Kr  5s1 4d 5 ; Re  Xe  6s 2 5d 5
Zr, Hf
3. Elements with d10 configuration: Nb, Ta
Mo, W
Cu : 4s1 3d10 Zn  4s1 3d10 9. Oxidation states:
Pd  5s 0 4d10 Ag : 5s1 4d10 Cd  5s 2 4d10 The transition metals exhibit a large number of
Au : 6s1 5d10 Hg  6s 2 5d10 oxidation states.
NISHITH Multimedia India (Pvt.) Ltd.,

With the exception of a few elements, most of


4. Highest MP in 3d serice : Cr
these show variable oxidation states.
4d serice : Mo
These different oxidation states are related to the
5d serice :W
electronic configuration of their atoms.
5. Enthalpy of atomisation is highest in 3d series : V
The existance of the transition elements in differ-
Enthalpy of atomisation is lowest in 3d series :
Zn ent oxidation states mean that their atoms can
6. Mercury is a liquid metal because Hg has lose different number of electrons.
This is due to the participation of inner (n - 1) d-
 Xe 4 f 14 5d 10 6s 2 configuration. Due to poor electrons in addition to outer ns-electrons be-
shielding nature of 4f and 5d orbitals, the 6s 2 cause, the energies of the ns and (n - 1) d-sub-
electrons are tightly held to the atom. This de- shell are nearly same.
creases the extent of delocalization of electrons For example, scandium has the outer electronic
and decrease the metal - metal bond strength. configuration 3d1 4s 2 .
7. SIZE OF ATOMS AND IONS It exhibits an oxidation state of +2 when it uses
The covalent radii of the elements decrease from both of its 4s-electrons for bonding but it can
left to right across a row in the transition series, also show oxidation state of +3 when it uses its
until near the end when the size increase slightly. two s-electrons and one d-electron.
On passing from left to right, extra protons are Similarly, the other atoms can show oxidation
placed in the nucleus and extra orbital electrons states equal to ns-and (n-1) d-electrons.
are added. The orbital electrons shield the nuclear It may be noted that the stability of a given oxi-
charge incompletely (d electrons shield less effi- dation state depends upon the nature of the ele-
ciently than p electrons, which in turn shield less ments with which the metal is combined.
effectively than s electrons). The highest oxidation states are found in com-
Because of this poor screening by d electrons, pounds of fluorides and oxides because fluorine
the nuclear charge attracts all of the electrons and oxygen are most electronegative elements.
more strongly; hence a contraction in size occure.
NISHITH Multimedia India (Pvt.) Ltd., 97
d-BLOCK & CO-ORDINATION COMPONDS JEE ADVANCED - VOL - VII
The examination of the common oxidation states For example, in MnO4 (Mn in +7 state) all the
exhibited by different transition metals reveals the
following bonds are covalent.
(i) The variable oxidation states of transition met- (v) Within a group, the maximum oxidation state
als are due to participation of inner (n-1) d and increase with atomic number.
outer ns-electrons. For example, iron (group 8) shows common
The lowest oxidation state corresponds to the oxidation states of +2 and +3 but ruthenium and
number of ns-electrons. osmium in the same group form compounds in
For example, in the first transition series, the low- the +4, +6 and +8 oxidation states.
(vi) transition metals also form compounds in low
est oxidation states of Cr  3d 5 4s1  and
oxidation states such as +1 and 0 or negative.
Cu  3d10 4s1  are +1 while for other, it is +2 The common examples are
 Ni  CO 4  ,  Fe  CO 5  in which nickel and
 3d 110
4s 2  .
(ii) Except scandium, the most common oxida- iron are in zero oxidation state.
tion state of the first row transition elements is (vii) The variability of oxidation states in transi-
+2 which arises due to loss of two 4s-electrons. tion elements arises because of incomplete filling
This mean that after scandium 3d-orbitals be- of the d-orbitals in such a way that their oxida-
come more stable and therefore, are lower in tion states differ by unity such as V II , V III , V IV
NISHITH Multimedia India (Pvt.) Ltd.,

energy than the 4s-orbitals. As a result, electrons


are first removed from 4s-orbitals. and V V .
(iii) The elements which show the greater num- This behaviour is in contrast with the variability
ber of oxidation states occur in or near the middle of oxidation states of non-transition elements (p-
of the series. block elements), where oxidation states normally
For example, in the first transition series, man- differ by a unit of two such as Sn 2 , In  , In 3 ,
ganese exhibits all the oxidation states from +2
to +7. etc.
The lesser number oxidation states in the begining (viii) Unlike p-block elements where the lower
of series can be due to the presence of smaller oxidation states are favoured by heavier mem-
number of electrons to lose or share (Sc, Ti). bers (due to inert pair effect), the higher oxida-
On the other hand, at the extreme right hand side tion states are more stable in heavier transition
end (Cu, Zn), lesser number of oxidation state is elements.
due to large number of d electrons so that only a For example, in group 6, Mo (VI) and W(VI)
fewer orbitals are available in which the electron are found to be more stable than Cr (VI). There-
can share with other for higher valence. fore, Cr (VI) in the form of dichromate in acidic
The highest oxidation state shown by any transi- medium is a strong oxidising agent where as
tion metal is +8.
MoO3 and WO3 are not.
(iv) In the +2 and +3 oxidation states, the bonds
formed are mostly ionic. The magnitude of ionization enthalpy gives the
In the compounds of higher oxidation states (gen- amount of energy required to remove electrons
erally formed with oxygen and fluorine), the bonds to form a particular oxidation state of the metal
are essentially covalent. in a compound. thus, the value of ionisation
Thus the bonds in +2 and +3 oxidation states enthalpies gives information regarding the ther-
are generally formed by the loss of two or three modynamic stability of the transition metal com-
electrons respectively while the bonds in higher pounds in different oxidation states.
oxidation states are formed by sharing of d-elec- Smaller the ionisation enthalpy of the metal, the
trons. stable is its compound.
98 NISHITH Multimedia India (Pvt.) Ltd.,
JEE
JEEADVANCED - VOL
MAINS - CW - VOL- VII
-I d-BLOCK & CO-ORDINATION COMPONDS
RELATIONS
For example, the first four ionisation enthalpies 11. Stability of the various oxidation states
of nickel and platinum are given below : Compounds are regarded as stable if they exist
Ionisation
at room temperature, are not oxidized by the air,
Ni Pt are not hydrolysed by water vapour and do not
enthalpies
IE1  IE2 2.49103 kJmol1 2.66103 kJmol1 disproportionate or decompose at normal tem-
IE3  IE4 8.80103 kJ mol1 6.70103 kJmol1 peratures.
Total 11.29103 kJ mol1 9.36 103 kJmol1 Within each of the transition Groups 3 - 12, there
is a difference in stability of the various oxidation
It is clear form the above table that the sum of
states that exist.
first two ionization enthalpies is less for nickel
In general, the second and third row elements
than for platinum.
exhibit higher coordination numbers, and their
Ni  Ni 2  2e  I.E.  2.49  103 kJ mol 1 higher oxidation states are more stable than the
Pt  Pt 2   2e  I.E.  2.66  103 kJ mol1 corresponding first row elements.
This gives the known oxides and halides of the
Therefore, ionization of nickel to Ni2  is ener- first, second and third row transition elements.
getically favourable as compared to that of plati- Stable oxidation states from oxides, fluorides,
num. chlorides, bromides and iodides.
Thus, the nickel (II) compounds are thermody- Strongly reducing states probably do not form
namically more stable than platinum (II) com-
NISHITH Multimedia India (Pvt.) Ltd.,

fluorides and/or oxides, but may as well form


pounds. the heavier halides.
On the other hand, the sum of first four icnisation
Conversely, strongly oxidizing states form oxides
enthalpies is less for platinum than for nickel as :
and fluorides, but not iodides.
Ni  Ni 4   2e  I.E.  11.29  103 kJ mol1 12. In 3d series:
Pt  Pt 4   2e  I.E.  9.36 103 kJ mol1 The element showing highest number of varible
oxidation states: Mn
Thus, the platinum (IV) compounds are relatively
The elements which does not show varible
more stable than nickel (IV) compounds.
oxidation states Sc,Zn(Sc: +3;Zn: +2)
Therefore, K 2 PtCl6 [having Pt (IV) is a well- 13. The stable highest oxidation state possibel in
known compound whereas the corresponding 3d series elements in their flourides is + 6 (Cr)
nickel compound is not known.
However, in solutions the stability of the com-
 CrF6 
pounds depends upon electrode potentials. 14. The +7 state for Mn is not represented in
10. Electrode potentials simple halides but MnO3 F is known,
0
Metals with E SRP  ve liberate H 2 from dil.HCl 15. The highest Mn fluoride is MnF4 whereas the
0
and those with ESRP  ve do not liberate. Zn, highest oxide is Mn 2O7
Fe, Mn displace H 2 from dil.HCl but Cu, Ag,
16. VO2  Cr2O72  MnO4 : oxidising power
Hg, Au do not displace.
Ti V Cr Mn Fe Co Ni Cu Zn
17. In 3d series Mn 3+ and Co3 ions are the
E0 2 -1.63 -1.18 -0.91 -0.44 -0.28 -0.25 -0.25 +0.34 -0.76
M /M strongest oxidizaing agents in aq. solution.
involts
E0 3+ -0.37 -0.26 -0.41 +1.57 +0.77 +1.97 18. Ti 2 , V 2 and Cr 2 are strong reducing
 
M /M2+
agents and will liberate hydrogen from dilute
(Involts )
acids
NISHITH Multimedia India (Pvt.) Ltd., 99
d-BLOCK & CO-ORDINATION COMPONDS JEE ADVANCED - VOL - VII
19. In 3d series, of the d 4 species , Cr 2 is This is because the nuclear charge is poorly
screened and so attracts all the electrons more
strongly reducing and Mn 3 is strongly re strongly.
ducing In addition, the extra electrons added occupy
inner orbitals.
Cr 2 to Cr 3 :d 2 to d 3 Consequently the denities of the transition met-
d 3
 half filled t 2g level  als are high.
Practically all have a density greater than
5 g cm 3 . (The only exceptions are Sc 3.0
Mn 3 to Mn 2  : d 4 to d 5 d 5
 half filled 
20. in 3d series , standard electrode potential g cm 3 and Y and Ti 4.5 g cm 3 .)
The densities of the second row are high and
M 2

/ M value is +ve for copper third row values are even higher.
The two elements with the highest densities are
21. Many copper (1) compounds are unstable in
aq solution and undergo disproportionation osmium 22.57 g cm 3 and iridium 22.61

2Cu   Cu 2  Cu g cm 3 .
0.15V 0.5V 28. Lowest density in 3d serics : Sc
Cu 2  Cu    Cu highest density in 3d serics : Ni, Cu
29. High density of post lanthanide elements:
NISHITH Multimedia India (Pvt.) Ltd.,

22. 2
Mn 2 compounds are more stable than Fe It is because of unexpectedly smaller size due to
towards oxidation to their +3 state lanthanide contraction.
23. Acid solution
r At.Wt d  g / cc 
0.56V 0.27V 0
MnO 4   MnO 42   MnO34 Ag 1.44 A 108 10.8
0.93V 0.1V 0
3
 MnO 2  Mn Au 1.44 A 196 19.4
0.2V 1.55V 30. REACTIVITY OF METALS
  Mn  OH 2  Mn
Many of the metals are sufficiently electroposi-
24. MnO 24 in acidic medium disproportionate to tive to react with mineral acids, liberating H 2 .
A few have low standard electrode potentials and
MnO4 and MnO2 remain unreactive or noble.
25. Acid solution: Noble character is favoured by high enthalpies
of sublimation, high ionization energies and low
1.33V 0.41V
Cr2O72  Cr 3  enthalpies of solvation.
0.91V
The high melting points indicate high heats of sub-
Cr 2  Cr limation.
The smaller atoms have higher ionization ener-
26. Basic solution
gies, but this is offset by small ions having high
0.13V solvation energies.
CrO 24  Cr  OH 3
This tendency to favour noble character is most
1.01V 1.4V
 Cr  OH 2    Cr pronounced for the platium metals (Ru, Rh, Pd,
Os, Ir, Pt) and gold.
27. DENSITY 31. The metals of the second and third transition se-
The atomic volumes of the transition elements ries have greater enthalpies of atomisation than
are low compared to elements in neighbouring the corresponding elements of the first transition
Groups 1 and 2. series.

100 NISHITH Multimedia India (Pvt.) Ltd.,


JEE
JEEADVANCED - VOL
MAINS - CW - VOL- VII
-I d-BLOCK & CO-ORDINATION COMPONDS
RELATIONS
32. Lattice structures of transition metals : bcc/ccp/ 34. COMMON COORDINATION NUM-
hcp (except Mn, Zn, Cd, Hg) BERS SHOWN BY TRANSITION ELE-
33. COLOUR PROPERTY MENTS OF FIRST ROW
1. Most of the compounds of transition metals i) Scandium : Sc3+ forms complexes with coor-
are coloured due to d - d transition. dination number of 6. examples of such com-
plexes are [Sc(H2O)6]3+, [ScF ]3- etc.
Ex : FeCl3 ,  Cu  NH 3  4  SO 4 etc 6
ii) Titanium : Ti4+ forms complexes with a co-
2. Polarization : NaCl, NaBr and NaI are all ordination number of 6. For example , [TiCl6]2-,
colourless; AgCl is also colourless. Thus the ha- 3
[Ti(SO4)3] 2- etc. Ti  H 2O 6  has purple
lide ions, Cl , Br  and I , and the metal ions

color.
Na  and Ag are typically colourless. iii) Vanadium : V2+ forms mostly octahedral
However, AgBr is pale yellow and AgI is yellow. complexes ( coordination number = 6), for ex-
The colour arises because the Ag  ion polarizes ample [ V(H2O)6]2+, K4[V(CN)6].7H2O. But
K4[V(CN)7].2H2O is also known with pentago-
the halide ions.
nal bipyramidal structure ( coordination number
This means that it distorts the electron cloud, and
= 7).
implies a greater covalent contribution.
V3+ Forms octahedral complexes such as
The polarizability of ions increases with size, thus
[V(H2O)6]3+
NISHITH Multimedia India (Pvt.) Ltd.,

I is the most polarized, and is the most V4+ is known to form square pyramidal com-
coloured. For the same reason, Ag 2 CO3 and plexes with a coordination number of 5. Example
of such complexes are [VOX4]2-, [VO(OX)2]2-
Ag 3 PO 4 are yellow, and Ag 2O and Ag 2S are (OX = oxalate), [VO(bipyridyl)2Cl]+etc
black iv) Chromium: Cr2+ forms octahedral com-
3. Some of the compounds of transition metals plexes, such as [Cr(H2O)6]2+ and [Cr(NH3)6]2+
are coloured due to “Charge transfer” and coordination number 6.
Ex: KMnO 4 , K 2Cr2 O7 , CrO3 , K 2 CrO 4 , Cu 2 O Cr3+ forms octahedral complexes, such as
etc [Cr(H2O)6]3+ and [Cr(H2O)5Cl]2+ with coordi-
nation number 6.
In MnO4 an electron is momentarily transferred v) Manganese :Mn2+ forms octahedral com-
from oxyzen to the matal, thus momentarily plexes, such as [MnCl6]4- and [Mn(en)3]2+ with
changing O 2 to O  and reducing the oxida- coordination number 6.
tion state of the metal from Mn3+ forms octahedral complexes, such as
K3[Mn(CN)6] with coordination number 6.
Mn  7  to Mn  6  Mn4+ forms octahedral complexes, such as
3 K2[MnF6] and K2[Mn(CN)6] with coordination
4. A dilute solutions of  Fe  H 2O 6  and number 6.
2
vi) Iron :
 Mn  H 2O 6  are colorless due to spin for- Fe2+ froms mostly octahedral complexes like
bidden transition. [Fe(H2O)6]2+ but few tetrahedral halides with
coordination number 4 like [FeX4]2- are also
known.
Fe3+ is known to form octahedral complexes such

as [Fe(H2O)6]3+. Tetrahedral :  FeCl4 

NISHITH Multimedia India (Pvt.) Ltd., 101


d-BLOCK & CO-ORDINATION COMPONDS JEE ADVANCED - VOL - VII
vii) Cobalt : V2 O 5 Converts SO 2 to SO3 in the Contact pro-
Co2+ is known to from both tetrahedral like
[Co(Cl) ]2- and octahedral such as [Co(H2O)6]2+ cess for making H 2SO 4
4
complexes. MnO 2 Used as a catalyst to decompose
Co3+ forms octahedral complexes. For example,
[Co(NH3)6]3+ and [Co(CN)6]3-. KClO3 has give O2
viii) Nickel Group : Fe/Mo Promoted iron is used in the Haber-
Ni2+ fo rms octahedral Bosch process for making NH3
2 2
 Ni  NH3 6  ,  Ni  H 2O 6  and square pla- FeCl 3 Used in the production of CCl4 from
2 CS2 and Cl2
nar  Ni  CN  4  , Ni  DMG  2 complexes.
2
FeSO 4 & H 2O 2 : Used as Fenton’s reagent
Few tetrahedral :  NiCl4  , trigonal bipyramidal for oxidizing alcohols to aldehydes.
and square based pyramidal structures are also PdCl 2 Wackers oxo process for converting
formed.
Pd2+ and Pt2+ are all square planar. C 2 H 4  H 2 O to CH 3 CHO
Ni3+ forms octahedral compounds. For example, Pd Used for hydrogenation (e.g. phenol to cy-
K3[NiF6]and [Ni(en)2Cl2]Cl. clohexanone).
NISHITH Multimedia India (Pvt.) Ltd.,

Pd4+ forms a few octahedral complexes like Pt/PtO2Adams catalyst, used for reductions.
[PdX6]2-, where X = F , Cl or Br. These are Pt Formerly used for SO 2  SO3 in the con-
generally reactive. Halide complexes are decom-
posed by hot water, giving [PdX4]2- and halo- tact process for making H 2SO 4
gen. In contrast Pt4+ forms large number of very Pt is increasingly being used in three stage-con-
stable octahedral complexes like [PtCl6]2-. vertors for cleaning car exhaust fumes.
ix) Copper, silver and gold Pt/Rh Formerly used in the Ostwald process
Cu+ forms tetrahedral complexes with Cl (for for making HNO3 to oxidize NH3 to NO
example, [Cu(Cl)4]3- and linear complexes like Cu Is used in the direct process for manufacture
-
[CuX2] . of (CH 3)2 SiCl2 used to make silicones.
Cu2+ forms complexes both of coordination num- Cu/V Oxidation of cyclohexanol/cyclohexanone
ber 4 ( like [CuX4]2- (X = Cl, Br, I) and of coor- mixtures to adipic acid which is used to make
dination number 6 nylon-66.
{like [Cu(en)3]2+, [Cu(H2O)3(NH3)3]2+}. CuCl2 Deacon process of making Cl2 from HCl.
Ag+, Au+ forms complexes with coordination Ni Raney nickel, numerous reduction pro-
number 2, [M(CN)2]-, [M(NH ) ]+1 cesses (e.g. manufact ure of
3 2
(M =Cu+1, Ag+1, Au+1 )
x) Zinc and Cadmium hexamethylenediamine, production of H 2 and
Zn2+ and Cd2+ forms both tetrahedral and octa- NH3 ,reducing anthraquinone to anthraquinol in
2
hedral complexes. For example, [MCl4 ] the production of H 2 O 2 ).
[M(NH3)2Cl2], [M(NH3)4] 2+, [M(H2O)6]2+etc. Ni Reppe synthesis (polymerization of alkynes)
 1
Hg forms  Hg  NH 3 2  ,  Hg  CN 2  e.g. to give benzene or cyclooctatetraene. com-
plexes
35. Catalytic Properties: Co2(CO)8 Hydroformylation catalyst
TiCl 4  Al  C2 H 5  3 Used as the Ziegler- RhCl(Pph 3 ) 3 Wilkinson catalyst. Stereo
selective and Stereospecific hydrogenation of
Natta catalyst in the production of
polythene. alkenes.
102 NISHITH Multimedia India (Pvt.) Ltd.,
JEE
JEEADVANCED - VOL
MAINS - CW - VOL- VII
-I d-BLOCK & CO-ORDINATION COMPONDS
RELATIONS

36. Al2 O3 or BaO do not form nonstoichiometric


CH 2  CH 2  CH 2
oxides. But FeO, VO, MnO can form
nonstoichiometric oxides as they can exhibit
NH 2 NH 2
variable valency.
37.

Ti 2 , V 2 ,Cr 2 :  cal  obs (iv) bn: butylenediamine

   magnetic moment  CH3  CH  CH  CH 3

Mn 2 , Fe 2 , Co 2 , Ni 2 ,Cu 2 :  cal   obs


H2 N NH 2
38. a)

basic amphoteric acidic


(v) OX 2  :oxalate
CrO Cr2O3 CrO3
VO V2 O3 , VO2 V2O5 O
O
MnO Mn2O3 , MnO2 Mn2O7
C
NISHITH Multimedia India (Pvt.) Ltd.,

b) Covalent character increases in the order: |


C
VCl2  VCl3  VCl4  VCl5
O O
FeCl2  FeCl3
(vi) acac  :acetylacetonate
c) TiCl4 is a liquid at room temperature.
CO-ORDINATION
1. LIGANDS :
O O
Bidentate: Two donations are accepted from
the ligand. For example, C C
(i) en: ethylenediamine
H 3C CH CH 3
CH 2  CH 2

H2 N NH2
(vii) dmg  : dimethylglyoximate

CH 3
(ii) pn: propylendiamine O
CN
CH3  CH  CH 2
CN
NH 2 NH 2
H 3C OH

(viii) gly  :glycinate


(iii) tn:trimethylenediamine

NISHITH Multimedia India (Pvt.) Ltd., 103


d-BLOCK & CO-ORDINATION COMPONDS JEE ADVANCED - VOL - VII

O As  CH3 2
CH 3  C
As  CH3 2
NH 2 O
c. Tridentate : Three donations are accepted from
the ligand. For example,
(ix) dipy: dipyridyl (i) dien: diethylrnetriamine

CH 2 CH 2
H2C
N N NH CH 2
H2N
NH 2
(x) o-phen: ortho-phenanthroline
(ii) imda 2 : iminodiacetate

NH
CH 2 CH 2
N
NISHITH Multimedia India (Pvt.) Ltd.,

N OC CO

O O
(xi) N, N’-Diethylthiocarbamate ion
d) Tetradentate: Four donations are accepted
S from the ligand. For example
(i) trien: triethylenetetraamine
Et 2 N  C
CH 2 CH 2
S
CH 2 NH CH 2
(xii) Salicylaldehyde anion
H2 N NH
H
C CH 2
O H2 N CH 2

O 3
(ii)  NTA  : nitrilotriacetate
(xiii) 8-Hydroxyquinolinol ion (oxine)

O CH 2 CH 2

C N C O

N O O
CH 2
O
C

(xiv) o-Phenylenebisdimethylarsine (diarsine) O O

104 NISHITH Multimedia India (Pvt.) Ltd.,


JEE
JEEADVANCED
MAINS - CW - VOL
- VOL- VII
-I d-BLOCK & CO-ORDINATION COMPONDS
RELATIONS
e. Pentadentate: Five donations are accepted  Examples of bridging ligands are:

from the ligand. For example,  EDTA : eth-


3 OH  , F ,Cl , NH 2 , CO, O 2 ,SO 42 etc.
3. Symmetrical and unsymmetrical bidentate
ylenediamine triacetate ligands:
 Bidentate ligands may be symmetrical or
CH 2  CH 2 CH 2
CH 2 unsymmetrical ligands.
N N CO  In symmetrical bidentate ligands the two
OC
H coordinating atoms(donor atoms) are the same.

CH 2 O Ex: en(N,N - donor atoms)
O
 In unsymmetrical bidentate ligands the two
C coordinating atoms are different
O O Ex: gly ( N , O - donor atoms)
4. Naming of the bridged ligands of the bridged
f. Hexadentate: Six donations are accepted from polynuclear complexes:
4  The complexes having two or more metal atoms
the ligand. For example,  EDTA  : ethylene- are called polynuclear complexes.
diamine tetracetate  In these complexes the bridging group is indicated
in the formula of the complex by separating it
CH 2 from the rest of the complex by hyphens and by
NISHITH Multimedia India (Pvt.) Ltd.,

CH 2 CH 2  CH 2
adding the prefix  before its name.
N N CO
OC  The greek letter  should be repeated before the
CH 2  name of each different bridging group
CH 2 O
O  Two or more bridging groups of the same kind
C are indicated by di-  -, tri-  - etc.
C
O 
O O Ex:
O
OH

g. Flexidentate: A ligand which shows variable  NH 3 5 Cr Cr  NH 3 5 Cl 5


1.
denticity is called a flexidentate ligand. For ex-
ample,   hydroxo-bis(pentaammine)chromium(III)
O chloride
O O O
4
S and S NH 2

 NH 3  4 Co Co  NH 3 4
O O O O 2. OH

(ii)  Co  NH 3  4 CO3  Br and Co  NH 3 5 CO3  Br


  amido-   hydroxooctaammine
CO3 is bidentate CO3 monodentate dicobalt(III) ion
2. Bridging ligands:
CO
 It is also possible that a monodentate ligand may
 C5 H 5  CO  Fe Fe  CO  C 5H 5 
have more than one free electron pairs and thus
3. CO
may simultaneously coordinte with two or more
atoms, i.e., the ligands forms two   bonds with
two metal atoms and thus acts as a bridge Di-   carbonyl-bis
between the metal atoms. (carbonylcyclopentadienyliron)
NISHITH Multimedia India (Pvt.) Ltd., 105
d-BLOCK & CO-ORDINATION COMPONDS JEE ADVANCED - VOL - VII

 CO 3 Fe  CO  2 Fe  CO 3  : Experimental weight loss agrees with theoretical


4.
wt.loss.
Tri-   carbonyl-bis(tricarbonyliron) (b) Linkage isomerism
2 2
5. Metal - to - metal bonding:  Pt ( NCS )4  3 and  Pt  SCN 4  3;
In complexes containing metal - to - metal bonds,  Mo  CN 8  and  Mo  NC 8 
the prefix bi - is used before the name of the (c) Coordination Isomerism:
metals forming a metal - to - metal bond. (a)
 CH 3 NH 2  4 Pt  Pt  NH 2 CH 3  4  [Co(en)3 ][Cr(C2 O 4 )3 ] and
 | | 
Ex:  Cl Cl  Cl 2 [Co(en)2 (C 2O 4 )][Cr(en)(C 2O 4 ) 2 ]
 
  [Cr(en) 2 (C 2O 4 )][Co(en)(C 2O 4 ) 2 ] and
dichlorooctakis(methylamine)bi-platinum(II) [Cr(en)3 ][Co(C 2O 4 )3 ]
chloride
(b) Co  NH 3 6  Cr  CN 6 
6.  -acid or  - acceptar ligands
There ligands not only donate the lone pair if and Cr  NH 3 6  Co  CN 6 
eleectrons to the central atom but also accept
the eelctron could from the central atom in (c)  Pt  NH 3  4  CuCl4 
NISHITH Multimedia India (Pvt.) Ltd.,

their low-lying vacant orbitals. This type of


back donation is known as “synergic effect” or ,  Cu  NH 3 4   PtCl4  and
synergic bonding
,  Pt  NH 3 3 Cl  Cu  NH 3  Cl3 
Ex: CO, CN  , NO  , PF3 , PR 3 C 2H 4 etc
Cr  NH 3 6  Cl3  K 3 Co  NO2 6 
 R  H,Et, Ph.... (d)
acetone    Cr  NH 3  6   Co  NO2 6   3KCl 
7. ISOMERISM    
(a) Hydrate Isomerism: Three isomers of
CrCl3.6H2O are known. From conductivity mea- Co  NH 3  6  Cl3  K 3 Cr  NO2 6 
surements and quantitative precipitation of the acetone
ionized chlorine, they have been given the fol-   Co  NH 3  6  Cr  NO2 6   3KCl 
lowing formulae: (e) GEOMETRICAL ISOMERISM
% Loss of H 2O
Complexes of type Ma 3b3 exists in two
1)[Cr(H2O)6]Cl3 violet -Nil-
geometrical forms which are named as
(three ionic chlorines)
facial(fac) and meri donial (mer) isomers
2)[Cr(H2O)5Cl]Cl2.H2O 6.75
green(two ionic chlorines) Ex:  Co  NO 2 3  NH 3 3 
3) [Cr(H2O)4Cl2].Cl.2H2O
dark green (one ionic chlorine) 13.50
a :When the complex is placed along with
A B
conc. H 2 SO4 , It does not show any weight loss.
A A A
Lattice water is removed and not the coordinated B
water. Mol.Wt of the complex : 166.5 M M
%Wt .Loss fo r one water molecule =
B A B A
18  100 B
 6.75% B
166.5 (fac) (mer)
106 NISHITH Multimedia India (Pvt.) Ltd.,
JEE
JEEADVANCED - VOL
MAINS - CW - VOL- VII
-I d-BLOCK & CO-ORDINATION COMPONDS
RELATIONS
n 0
The complexes of the  M  AA 2  can  2 
Pd NH  CH  CH   CH  CH   CO  
3 3 2
2

show geometrical isomerism if the can have four geometrical isomers
ligand AA has a chiral centre. For
Me Me
example, consider the H 2N NH 2
2 C C
H Pd 2  H
gerometrical isomers of  Pt  bn  2  , |
C
|
C
O O O
where O

Optically inactive (Plane of symmerty)


bn:CH 3  CH  NH 2   CH  NH 2   CH3 (cis)

Me Me
N N H H2N O
O
2 C C
Pt
Me | Pd 2  | H
N N C C
Me Me O O NH 2 Me
Opticallyinactive
Optically active (trans)

Me
H H2 N
NH2
Me Me
C C
NISHITH Multimedia India (Pvt.) Ltd.,

2 H2 N NH 2
Me Pt H
C C C C
N O H | Pd 2  | H
O O C C
Opticallyinactive  cis  O O O
O
Optically inactive (Plane of symmerty)
Me Me (cis)
N N
2
Pt
Me H2N O
N N O
Me Me C C
Opticallyinactive H | Pd 2  | H
C C
O O NH 2 Me
Me Me
N N
Optically inactive (Centre of symmerty)
Pt 2 (trans)

N N
Me Me (viii) Geometrical isomers are possible for
Opticallyinactive square planar binuclear complexes of the
type
Me Me
N N  Pd 2 Cl 4  PPh 3 2 
2
Pt
N N
Me Me
Cl Cl Cl Cl Cl Ph 3 P
Opticallyinactive

n Pd Pd Pd Pd
Vii) The complexs of the type  M  A B  2 
can show gemetric as well as, optical isomer Ph 3 P Cl Ph 3 P Ph 3 P Cl Cl
ism if the ligand AB has a chiral centre. For
cis trans
example , the compound
NISHITH Multimedia India (Pvt.) Ltd., 107
d-BLOCK & CO-ORDINATION COMPONDS JEE ADVANCED - VOL - VII
7. Crystal field theory (CFT) : The total crystal fiels stabilization energy is
According to CFT, the bonding in complex giveny by
ions is purely electrostatic CFSE  Octahedral   0.4x
This theory regards the ligand atoms of ionic  t 2g   0.6x  eg 
ligends such as F , Cl or CN  as negative
Where x  t 2g  and x  eg  are the number of
point charges (also called point charges)
If the ligand molecules are netral, those are electron occupying the t 2g and eg orbitals
regarded as point dipoles or simply dipoles,
the negative end pointing towards central respectively
metal ion. The CFSE is zero for ions with d 0 and d10
The complex is regarded as a combination of
confifiguration in both strong and weak ligand
a central metal ion surrounded by ligands
fileds
which acts as point charges or point dipole.
The arrangement of ligands around the central The CFSE is also zero for d5 configurations
metal ion or atom is such that the repulsion in a weak field
between these negative points or dipoles is All the other arrangements have some CFSE,
minimum. which increases the thermodynamic stability
In a free transition metal or ion, these are five of the complexes
NISHITH Multimedia India (Pvt.) Ltd.,

d orbitals which are designated as


9. In d 2 sp 3 or sp 3 d 2 the orbitals used in
d xy , d yz , d zx , d x 2  y 2 and d z2 . The five d-
hybridization are d x 2  y 2 and d z 2 .
orbitals are divided into different sets de
pending on the nature of their orientation in 10. Ligand Field Splitting in Square Planar Com-
space. 2
plex:  PtCl4  , Pt 2  : 5d 8
8. Ligand Field Splitting in Octahedral
Complexes:
d x2  y 2

d xy

d z2

d xz d yz

The d-orbital used in dsp 2 hybridisation is


d x2  y2 .
11. Ligand Field Splitting in Tetrahedral Com-
In octahedral complexes , the filling of t 2g plexes:
orbitals decreases the energy of a complexs
that makes it more stable by 0.4 0 perelec-
tron. Filling eg orbitals increases the en -
ergy by 0.6 0 per electron
108 NISHITH Multimedia India (Pvt.) Ltd.,
JEE
JEEADVANCED - VOL
MAINS - CW - VOL- VII
-I d-BLOCK & CO-ORDINATION COMPONDS
RELATIONS
12. Factors influencing the magnitude of  o : Spectrochemical series:
I. Nature of the metal cation: I  Br   Cl  SCN   N 3
a) Among the cations from atoms of the same
transition series cations with a higher oxidation   C2 H5O 2 PS2  F   NH2 2 CO
states has a larger value of  o than that of lower
 OH   C2O 42  H 2O  NCS  H 
oxidation states, because the central ion with
higher oxidation state will polarise the ligands  CN   NH 2CH 2COO 
more effectively and thus the ligands would
approach such cation more closely.  NH3  C5H 5 N  en  SO32
Ex:
 NH 2OH  NO 2  Phen  CH 3
2
 0 for  Fe II  H 2 O 6   10, 400 cm 1  3d 6 
 CN   CO
3
 0 for  Fe III  H 2 O 6   13, 700 cm 1  3d 5  This order independent of the nature of the central
metal ion and the geometry of the complex.
b) Among the cations having same number of ‘d’ III. Geometry of the complex:
electrons and the same geometry of the complex, As the geometry changes  value chages.
cations with higher oxidation state has large value
sp   o   t
NISHITH Multimedia India (Pvt.) Ltd.,

of  o .
Ex: 4
sp  o
 0 for  V II  H 2 O 6 
2
 12, 400 cm 1  3d 3  3

3 4
 0 for  Cr III  H 2 O  6   17, 400 cm 1  3d 3  t  o
9
c) In case of complexes having the cations with 13. ORGANO METALLIC COMPOUNDS
the same charges but with different number of EXAMPLES
‘d’ electrons in the central metal cation, the a) Grignard Reagent R –Mg –X where R is a alkyl
magnitude of  o decreases with the increase of or aryl group and X is halogen
the number of ‘d’ electrons. b)  CH 3 4 Sn,  C 2 H 5 4 Pb, Al 2  C H 3 6 Al 2  C 2 H 5 6 etc
Ex:
2
 0 for  CO II  H 2 O 6   9,300 cm 1  3d 7  Bridging groups
2 H3C CH3 CH3
 0 for  Ni II  H 2 O  6   8,500 cm 1  3d 8 
Al
Al
d) As the quantum number of the ‘d’ orbitals of
H3C CH3
the central atom increases  o value increases. CH3

Ex: ii) pie (p) bonded organometallic compounds :


3 These are the compounds of metal with alkenes,
 0 for CO III  NH 3 6   23, 000 cm 1  3d 6 
alkynes, benzene and other ring compounds.
3 e.g. Zeise’s salt
 0 for  Rh III  NH 3 6   34, 000 cm 1  4d 6 
II. Strong/weak ligands:  
K  PtCl3 2  C2 H 4 
 
Strong ligands possess higher  o values.
NISHITH Multimedia India (Pvt.) Ltd., 109
d-BLOCK & CO-ORDINATION COMPONDS JEE ADVANCED - VOL - VII
of these vacant anti - bonding molecular orbitals
H H
with a filled metal orbital of  - symmetry results
in the formation of a second bond called dative
C
K
+ bond. The dative bond serves to pass the
Cl
C excessive negative charge(that accumulates on
H
Pt H the metal due to the formation of   bond) to
Cl Cl the ligand.
 Back donation of electrons from the metal to the
ligands is possible only if the metal is in a low
Potassium trichloro ( 2 -ethylene)platinate(II) oxidation state. Hence, CO ligand stabilizes low
oxidation states of metals in its complexes.
Has both  and  ligands.
 The   and   bonding mutually supplement
Ferrocene 
Fe 5  C5 H 5  2
each other. The donation of electrons to metal,
during the formation of  bond, increases the
partial positive charge on the CO ligand, making
-1 it a better acceptor. The back donation of metal
electrons to the ligands increases the partial
Fe + 2
negative charge on CO, making it, in effect, a
-1 better donor. This mutual reinforcement is called
synergism. As a result of synergism,   bonding
strengthens   bonding vice versa.
NISHITH Multimedia India (Pvt.) Ltd.,

bis ( 5 -cyclopentadienyl)iron(II)  A consequence of synergism is that CO ligand


Sandwich complex forms a large number of complexes with transition
metals in low oxidation states, even though it is a
bis (benzene) chromium (0), Cr    C6 H 6 2
6
poor Lewis base for the other species. Another
consequence of synergism is that it increases the
bond order of
metal - carbon bond and decreases the bond
Cr order of carbon - oxygen bond. This results in
a) shortening of metal - carbon bond as
compared to the metal - carbon single bond.
b) lengthening of carbon - oxygen bond as
sandwich complex compared to carbon - oxygen bond.
iii)  &  bonded organometallic comp.  Metal carbonyls exhibit a strong tendency to
eg Metal carbonyls achieve Sidgwick EAN values and as a result
14. Metal carbonyls: 0
1)  Mn  CO 6  can act as a reducing agent. The
 The highest occupied molecular orbital of carbon
complex loses an electron to attain the noble gas
monoxide, the  2p Z orbital, holds two configuration and hence obey Sidgwick EAN
electrons. This electron pair is loosely held and rule.
hence is available for donation to metal. The e 0  
 Mn  CO 6    Mn  CO 6 
bo nding takes place when the filled
 EAN  37   EAN  36 
 2p Z molecular orbital of CO overlaps with an 0
2)  V  CO 6  can act as oxidising agent. The
empty orbital of the metal. As a result, a normal
ligand - to - metal  bond is formed. complex gains an electron to attain the noble gas
CO also has two degenerate, empty, anti - configuration and hece obey Sidgwick EAN rule.

bodning  orbitals. These are the lowest 0
e
 V  CO  6  

 V  CO 6 

unoccupied molecular orbitsls of CO, mutually  EAN  35   EAN 36 


perpendicular to each other. The overlap of one
110 NISHITH Multimedia India (Pvt.) Ltd.,
JEE
JEEADVANCED - VOL
MAINS - CW - VOL- VII
-I d-BLOCK & CO-ORDINATION COMPONDS
RELATIONS

3)
0
 Mn  CO 5  undergoes dimerisation to attain II. Irving - Willium order:
Stabilities of high spin complexes of the ions
the noble gas configuration and hence obey
between Mn 2 and Zn 2 with a given ligand
Sidgwick EAN rule.
0
vary in the order:
2  Mn  CO 5    Mn 2  CO 10 
 EAN  35   EAN 36 
Mn 2  Fe 2   Co 2  Ni 2   Cu 2   Zn 2
Anionic carbonyl complexes are called Radii of these ions are in the order:

carbonylate ions, these are also referred to as Mn 2  0.91A0   Fe 2   0.83A0 
carbometallates.
Ex:  Co 2   0.82A0   Ni 2   0.78A0 
2 
1)  Ti  CO 6  ,  V  CO 6  : Isoelectronic,
 Cu 2   0.69A0   Zn 2  0.74A0 
isostructural with  Cr  CO 6  . III. Electronegativity of the eentral metal ion:
3 2 A cental metal ion with higher electronegativity
2)  V  CO 5  , Cr  CO 5  ,
would give the most stable complexes.

 Mn  CO 5  : Isoelctronic, isostructural with IV. Size and charge of ligand:
For charged ligands, the higher the charge carried
NISHITH Multimedia India (Pvt.) Ltd.,

 Fe  CO 5  by them and the smaller their size, the more stable


4 3 are the complexes formed
3)  Cr  CO 4  ,  Mn  CO 4  , (for class a metals)
2  (Class a metals: Sc, Ti, V, Cr etc)
 Fe  CO  4  ,  Co  CO  4  : Isoelectronic,
2 2

isostructural with  Ni  CO 4  Ex: Stability of


 FeF   FeCl
log 10  6  log  20
 As the negative charge on the coordination
V. Basic character of the ligands:
sphere of a carbonyl complex increases, the
extent of back bonding also increases. This, in The more basic is the ligand, more easily it can
turn, leads to an increase in metal - carbon bond donate electron pairs to the central ion and hence
order and a decrease in carbon - oxygen bond more easily it can form complexes of greater
order. stability.
Ex: In VI. Chelating effect:
2 2
 V  CO 5  , Cr  CO 5  ,  Mn  CO 5 

 Complexes containing chelate rings are usually
 I  II   III  more stable than similar complexes containing no
Carbon - oxygen bond order: I < II < III rings,i.e., multidentate ligands ingenral form more
Carbon - oxygen bond length: I > II > III stable complexes than the monodentate ligands.
Carbon - oxygen bond strength: I < II < III 2 2
Metal - carbon bond strength: I > II > III  Ni  en 3    Ni  NH3  6 
Ex: Stability:   aq   aq 
15. Factors affecting the stability of complexes:  log 18.1  log  7.99 
I. Charge and size of the metal ion:  The greater the amount of disorder produced in
Ingeneral the metal ion with larger charge and the products during the reaction relative to the
smaller size form more stable complexes.
reactants, the greater the increase in entropy
3 4
during the reaction and hence the greater the
Stability of:  Fe  CN 6    Fe  CN 6 
 
 log  31  log 8.3 stability of the products.
NISHITH Multimedia India (Pvt.) Ltd., 111
d-BLOCK & CO-ORDINATION COMPONDS JEE ADVANCED - VOL - VII
Ex: When ethylene diamine(en) is allowed to react LEVEL -V
2
with hydrated complex ion,  M  H 2 O  6  , it SINGLE ANSWER QUESTIONS
replaces two H 2O molecules from it, since it is a
1. Which of the following statements is
bidentate ligand. incorrect?
This process increases the number of particles in
(A) Mercurous ion exist as Hg 
the system and hence its disorder and entropy.
(B) Mercurous ion is diamagnetic and exist as
2
Thus the complex  M  H 2 O  4  en   is more dimer Hg 22 
2 (C)Mercurous ion is colourless
stable than  M  H 2 O  6 
(D) There is a covalent bond between two Hg 
2
 M  H 2 O 6   en  ions
2. The metal that has the highest melting point
2 and used in making hard steel is
 M  H 2 O 4  en    2H 2O (A)Cu (B) Mn (C) Zn (D) W
3. The property, which is not charactestics of
NISHITH Multimedia India (Pvt.) Ltd.,

VII. Chelate ring size:


transition metals
 The large the number of the chelate rings in a (A) varible oxidation states
complex, the greater is its stability. (B) tendency to form complexes
 The stability of the complexes also depends on (C) formation of coloured compounds
the number of the atoms present in the rign. (D) They are usually diamagnetic
4. The pair that has the greatest malleability
 The chelates containing 3 - membered ring
and ductility property is
including the metal are very unstable.
(A) Na, K (B) Pb, Sn
 4- membered chelate rings are rare and occur in (C) Zn, Mn (D) Cu, Au
carbonate, nitrate and sulphate chelates. 5. The metal that has the lowest boiling point
 5-membered chelates are frequently more stable among the following is
than the 6-membered chelates when the atoms (A) Ti (B) Zn (C) Cu (D) Fe
in the rign are joined by single bonds only. 6. In a transition series, as the atomic number
increases, paramagnetism
 6-membered chelates are more stable than
(A) increases gradually
5-membered chelates of heterocyclic ligands or (B) decreases gradually
of ligands involving conjugation in the chelate ring. (C) first increases to a maximum and then de-
VIII. Steric effects: creases
When a bulky group is either attached to or (D) first decreases to a minimum and then in-
creases
present near the donor atoms of a ligand, mutual
7. Oxide of metal cation which is not
repulsion among the ligand occurs and
amphoteric
consequently the metal - ligand bond is
(A) Al3 (B) Cr 3 (C) Fe3 (D) Zn 2 
weakened. Thus large bulky ligands form less
stable complexes than do the analogous smaller 8. The most abundant transition metal in earth
crust is:
ligands.
A) Zn B) Fe C) Hg D) Au
112 NISHITH Multimedia India (Pvt.) Ltd.,
JEE
JEEADVANCED - VOL
MAINS - CW - VOL- VII
-I d-BLOCK & CO-ORDINATION COMPONDS
RELATIONS
9. The electron which take part in order to ex-
hibit variable oxidation states by transition
15. MnO 4 is intense pink colour, though Mn
metals are is (+7) oxidation state , It is due to
A) ns only A) oxygen gives colour to it
B)  n  1 d only B) charge transfer when oxygen gives its elec-
C) ns and  n  1 d only but not np tron to oxygen
C)Charge transfer when oxygen gives its elec-
D)  n  1 d and np only but not ns
10. Zn and Cd metals do not show varible va- tron to Mn making it Mn   VI  hence coloured
lency because: D) None of these
A) They have only two electrons in the outer-
16. Cementite is:
most subshells
B) their d-subshells are completely filled A) Interstitial compound of iron and carbon
C) their d-subshells are partially filled ` B) An alloy of Fe and Cr
D) they are relative soft metals C) A compound resembling cement
11. Each of the following ion contains vanadium
the +5 oxidation state except D) An ore of iron
 17. The radii (metalic) of Fe,Co and Ni are
A) VO 2 B) V  OH 4 nearly same
2 This is due to
C) VO 2 D)  VO3 OH 
NISHITH Multimedia India (Pvt.) Ltd.,

12. Paramagnetism is not exhibited by : (A)lanthanide contraction


A) CuSO 4 .5H 2O B) CuCl 2 .5H 2O (B) increase in radii due to increase in ‘n’ is com-
pensated by decrease in raddi due to
C) CuI D) NiSO 4 .6H 2O
increase in effective nuclear charge(Z)
13. The wrongly statement regarding transition
metals among the following is (C) Decrease in raddi due to increasing screeing
A) 4s electrons penetrates towads the nucleus effect is compensated by increase in size due to
more than 3d electrons increasing effective nuclear charge
B) Atomic radii of transition metals increase rap- (D) Atomic radii do not remain constant but de-
idly with increase in atomic number because of
crease in a normal gradation
poor shielding of nuclear
attraction by (n-1) d electrons 18. VO 4 , CrO 42 and MnO 4 are pale yellow ,
C) second and third transition series
elements have nearly the same size strong yellow and intense purple respec-
D) their densities are higher and densities of the tively in aqeous solution. The darkening of
5d series elementrs are higher than those of 4d colour is due to
series elements (A) charge transfer
14. Ionisation energies of Ni and Pt in kJ mol1 (B) d  d transition
are given below
(C) half-filled d-sub -shells
 IE 1   IE 2  IE 3   IE 4 (D) increasing number of unpaired electrons
19. Metals which do not form amalgam are
Ni  2.49 8.80
Pt  2.60 6.70 (A) Fe (B) Zn (C) Ni (D) Au
So, (select the correct statement ) 20. The hybridization of the metal in  CoF6 
3

A) nickel (II) compounds tend to be thermody-


namically more stable than platinum (II) is
B) Platinum (IV) compounds tend to be more
stable than nickel (IV) (A) sp 3 d 2 (B) d 2 sp 3
C) (A) & (B) Both (C) dsp 3 (D) sp 3 d
D) none of these
NISHITH Multimedia India (Pvt.) Ltd., 113
d-BLOCK & CO-ORDINATION COMPONDS JEE ADVANCED - VOL - VII

21. The hybridization of Cr in Cr  en 3  is


3 29. Which of the following statements is correct?
3 3
(A) CoF6  and Co  NH3 6  both are para-
(A) d 2 sp3 (B) sp 3d (C) sp 3d 2 (D) dsp 3
22. The magnetic moment of the complex magnetic complexes
3 3 3
Ti  H 2O  6  is (B) CoF6  and Co NH3 6  both are high

(A) 3.87 BM (B) 1.73 BM spin complexes


3 3
(C) 2.84 BM (D) 5.87 BM (C) CoF6  is octahedral while Co NH3 6 
23. Which one of the following coordination num- has a pentagonal pyramid shape
bers can show a square pyramidal geometry 3
? (D) CoF6  is outer orbial complex while
(A) 4 (B) 3 (C) 7 (D) 5 3
 Co  NH 3  
 6
is inner orbital complex
2
24. If the formation constant of Cu  NH 3 4  30. What is wrong about the compound
is 2 103 , then its dissociation constant is

K Pt 2  C2H4 Cl3  ?
  
(A) 5 104 (B) 2  103
(A) It is called Zeise’s salt
(C) 5 102 (D) 0.2  103 (B) It is only   bonded complex
The IUPAC name of  Fe  H 2O  6  Cl3 is (C) Oxidation number of Pt is +2
NISHITH Multimedia India (Pvt.) Ltd.,

25.
(A) Hexaaquiron(III) chloride (D) Four ligands surrounds the Platinum atom.
(B) Hex aquoiron(III) trichloride 31. Which species is not available in Fenton's
(C) Hexaaquairon(III) chloride reagent.
(D) Hexa aquairon(III) trichloride a) Fe+2 b) HO.
26. IUPAC name of Co  en 2 Cl2  Cl is c) HO– d) HO+
(A) dichloridobis (ethylene diammine) cobalt (III) 32. Zinc gives H 2 gas with H 2 SO4 and conc
Chloride. HCl but not with conc. HNO3 because.
(B) dichloridobis (ethylenediammine) cobalt(III)
Chloride
(C) dichlorobis (ethylene diamine) cobalt(II) a) NO3 ion is reduced in preference to
Chloride. hydronium ion
(D) dichloridobis (ethylene diamine) cobalt(III)
chloride b) Conc. HNO3 is a weaker acid than conc.
27. Which is not true statement ? H 2 SO4 and conc HCl
(A) Ions of d-block elements are coloured due
c) Conc. HNO3 acts as a reducing agent
to d—d transition.
(B) Ions of f-block elements are coloured due d) Zinc is more reactive than H 2
to f—f transition.
33. Which complex is likely to show optical
( C ) [Sc(H 2 O) 6 ]3 and[Ti(H 2 O) 6 ]4  a r e
coloured complexes activity
(D) Cu  is colourless ion. A) Trans - [Co (NH3)4Cl2]+
28. Among the following ions, which one has the
highest paramagnetism? B) [Cr (H2O)6] 3+
3 2 C) Cis - [Co(NH3)2 (en)2]3+
(A) Cr H2O 6  (B) Fe H2O 6 
2 2
D) Trans - [Co (NH3)2(en)2]3+
(C) Cu H2 O 6  (D)  Zn H2O 6 
114 NISHITH Multimedia India (Pvt.) Ltd.,
JEE
JEEADVANCED - VOL
MAINS - CW - VOL- VII
-I d-BLOCK & CO-ORDINATION COMPONDS
RELATIONS
34. Which of the following complex ion has a 42. The ratio of magnetic moments of Fe (III)
magnetic moment same as that of and Co(II) is :
[Cr(H2O)6]3+? a) 5: 7 b) 35 : 15
A) [Mn (H2O)6 ]4+ B) [Mn (H2O)6] 3+ c) 7 : 3 d) 24 : 15
C) [Fe (H2O)6] 3+ D) [Cu (NH3)4] 2+ 43. Which of the following statement is incor-
35. Ammonium dichromate is used in some rect ?
fireworks. The green coloured powder blown a) The electronic configuration of Cr is
in the air is  Ar  3d 5 , 4S 1
a) CrO3 b) Cr2 O3
b) The magnetic quantum no.may have negative
c) Cr d) CrO  O2  value
36. Identify the incorrect sttement among the c) In silver atom, 23 electrons have a spin of one
following : type and 24 of the opposite type
a) misch metal is a pure lanthanide metal
d) The oxidation state of nitrogen in HN3 is -3.
b) Magnetic moment increases from Ce(4f1
5d16s2) to Pr(4f36s2) in their +3 oxidation state. 44. Pick out the correct option. Where T
c) In lanthanides from Ce+3 to Lu+3 there is a regu- stands for true. F stands for false.
lar decrease in size I) The energy of the 3d orbital is high when
d) Ce+4 is a powerful oxidizing agent compared to 4s orbital in hydrogen atom
NISHITH Multimedia India (Pvt.) Ltd.,

37. For which one of the follwoing ions, the colour


is not due to a d - d transition ? II) The electron density in xy plane in d x 2 - y 2
2
a) CrO42  b) Cu  NH 3  4 orbital is zero
3
III) 24th electron in Cr goes to 3d orbital
c) Ti  H 2 O 6 d) CoF63 IV) The three quantum numbers were clearly
38. Gold and platlnum dissolves in aquaregia to explained in terms of schrodinger wave equa-
produce respectively tion
a) H  AuCl4  and H  PtCl3  a) TTFF b) TFTT
c) TFTF d) FFTT
b) H  AuCl4  and H 2  PtCl6 
c) H 2  AuCl6  and H  PtCl4 
MULTIPLE ANSWER QUESTIONS
d) H 2  AuCl6  and H  PtCl6 
39. AgCl on fusion with Na2CO3 forms: 45. The unstable compounds are
a) Ag2CO3 b) Ag2O
c) Ag d) Ag2C2 (A) MnI 7 (B) CrO3 (C) Mn2O7 (D) CrI 6
40. CrO3 dissolves in aqueous NaOH to give : 46. Which one of the following reactions cannot
a) Cr2O72 b) CrO42 occur ?
c) Cr OH 3 d) Cr OH 2 (A) Cu  ZnSO4  CuSO4  Zn 
41. In acidic medium (conc.H2SO4) (B) Cu  2 AgNO3  Cu  NO3  2  2 Ag 
Hydrogen peroxide reacts with
K2Cr2O7 in the presence of ether (C) Cu  FeSO4  CuSO4  Fe 
forms a deep voilet coloured chro
mium peroxide. The oxidation (D) 3Ag  AuCl3  3AgCl  Au 
state of chromium in chromium 47. Which of these are liquids at room tempera-
peroxide is . ture ?
a) +2 b) +3
c) +5 d) +6 (A) TiCl4 (B) Zn (C) Hg (D) CuCl2

NISHITH Multimedia India (Pvt.) Ltd., 115


d-BLOCK & CO-ORDINATION COMPONDS JEE ADVANCED - VOL - VII
48. The typical acidic oxides are 56 In which of the following compounds (s), the
(A) MnO (B) Mn2O7 (C) CrO (D) CrO3 colour is due to the charge transfer spectra
49. The compounds that undergo hydrolysis (A) KMnO 4 (B) CrO3
readily is/are
(C) CuCl 2 (D) Cu 2O
(A) TiCl4 (B) VCl5 (C) FeCl3 (D) CaCl2 57. Transition elements act as good catalysts
because
50. Which are the best suitable as coinage met- (A) Presence of partially filled ‘d’ orbitals
als ? (B) Form H-bonding easily
(A) Mg (B) Cu (C) Ag (D) Au (C) Transition elements show variable
51. Order of paramagnetic character among fol- oxidation state
lowing elements is/are
(D) Easy interconvertibility of oxidation states due
(A) Mn>Fe>Cr (B) Fe >Zn>Cr
to low oxidation and reduction potential
(C) Cr>Fe>Zn (D) Cr>Mn>Fe
52 Which of the following statements are cor-
rect? 58. The true statemens among the following are:
(A)Transition elements exhibit higher (A) Cu 2 undergoes disproportionation in aq
enthalpies of atomization as they have stronger solutions
interatomic interaction (B) All Cu(II) salts are known except the
IE 2 of 23 V  24 Cr 25 Mn iodide
NISHITH Multimedia India (Pvt.) Ltd.,

(B) (C) The only transition metal in 3d series with a


and 28 Ni 29 Cu 30 Zn
(C) Ni(II) compounds are more stable than Pt(II)
0

2

positive E M / M value is copper
where as Pt (IV) compounds are more stable (D) Copper has the highest second ionization en-
than nickel (IV) thalpy among all the 3d elements
(D)The elements which gives the greatest num-
59. Which of the following statements is/are
ber of oxidation states does not occur near the
true?
middle of the series
53. The diamagnetic compounds is/are (A) In metal carbonyl complexes
dC–Oincreases compared to that in CO molecule
(A) HgCl2 (B) Hg 2Cl2
(C) Cu2Cl2 (D) K 2 Cr2 O7 (B) The pair of compounds [Cr(H2O)6] Cl3 and
54. The colour of the transition metal ions/is [CrCl3(H2O)3] . 3H2O show hydrate
due to isomerism
A) d- d transition of electrons in presence of (C) dZ2 orbital of central metal atom/ion is used
ligands
in dsp2 hybridisation
B) charge transfer from ligands to metal ion
(D) Facial and meridional isomers associated with
C) change in the geometry
D) polarisation of anion by cation [Ma3b3] n type complex compound, both are
55 Select the correct statement(s) wih respect optically inactive
to oxides and oxoanions of transition metals 60. A d-block element forms octahedral complex
(A) Among oxides of chromium CrO is basic but its magnetic moment remains same ei-
ther in strong field or weak field ligand.
, Cr2O 3 amphoteric and Cr2O 3 is acidic
Which of the following is/are correct?
(B) No higher oxides of iron above Fe 2O 3 are (A) d-block element always forms colourless
found compound
(C) Ti ,V, Cr and Mn from oxides MO and their (B) Number of electrons in t2g orbitals are higher
correct increasing order of acidic character is than in eg orbitals
MnO<CrO<VO <TiO (C) It can have either d3 or d8 configuration
(D) Vandium (V) oxide does not react with acids (D) It can have either d7 or d8 configuration
but reacts with alkalies only
116 NISHITH Multimedia India (Pvt.) Ltd.,
JEE
JEEADVANCED - VOL
MAINS - CW - VOL- VII
-I d-BLOCK & CO-ORDINATION COMPONDS
RELATIONS
61. The complexes that have a magnetic mo- 70. Which of the following are cis isomers ?
ment of 1.73 BM is
3
a
4
(A) Ti  H 2O  6  (B) V  H 2O  6  b b
a b
2 3
(C)  Mn  H 2 O 6  (D)  Mn  H 2 O 6  M
(A) M (B)
62. The square planar complexes are
b a
2 b a
(A)  Pt  NH 3 2 Cl2  (B)  Ni  CN 4  b
2 2
(C)  NiCl4  (D)  PtCl4 
63. The tetrahedral complexes are a b
1 b b a b
(A) Ni  CO 4 (B)  FeCl4 

(C)  MnCl4 
2
(D)  CoCl4 
2 M M
(C) D)
64. The diamagnetic complexes are a b a b
3
(A) Cr  CO 6  (B) Co  NH 3 6  b a
NISHITH Multimedia India (Pvt.) Ltd.,

4 2
(C)  Fe  CN 6  (D)  Ni  NH 3 6  71. The following complexes are given
1) trans - [Co(NH 3 ) 2 Cl2 ]
65. The strong field ligands or moderate ligand
are 2)cis- [Co NH3 2  en 2 ]3 
(A) NH 3 (B) en
3) trans - [Co NH3 2  en 2 ]3 
(C) CO (D) CN 
4) NiCl42-
66. d  d transition is possible in 5) TiF62-
(A) Cu  NH 3 4 
2
(B) KMnO4 6) CoF63-
Choose the correct code
(C) K 2Cr2O7 (D) Co  NH 3 6 
3 (A) (1), (2) are optically active,
(B) (2) is optically active, (1),(3) are optically
67. Which of the following can act as bridging inactive
ligands ? (C) (4), (6) are colourless and (5) is coloured
(A) OH  (B) H 2O (D) (4) is coloured and (5) is colourless
(C) CO (D) Cl  72. Among the following which is most stable ?
3
68. Which of the following can give rise to link- (1)  Fe  CN 6  and  FeF6 3
age isomerism ? (X) (Y)
(A) en (B) NO2  3 4
(2)  Fe  CN 6  and  Fe  CN 6 
(C) CN  (D) SCN 
(S) (T)
69. Which of the following can show geometri-
cal isomers( M= metal,a,b,c - monodentate (3) [Cr  CN 6 ]3 and [Cr  CN 6 ]4
ligands ? (P) (Q)
(A) Ma3b (B) Ma4b2 Choose the correct code
(A) 1-X, 2-T, 3-Q (B) 1-X , 2-S, 3-Q
(C) Ma2bc ( squareplanar ) (D) Ma2b2c2 (C) 1-X, 2-S, 3-P (D) 1-Y, 2-T. 3-Q
NISHITH Multimedia India (Pvt.) Ltd., 117
d-BLOCK & CO-ORDINATION COMPONDS JEE ADVANCED - VOL - VII
73. Which of the following is /are correct? onwards, this character decreases as electrons
(A) Ligands with filled  orbitals are called  get paired up. The paramagnetic behavior is
donors expressed in terms of magnetic moment which is
(B)  donor ligand forms sigma bond with metal because of the spin of unpaired electrons (n). It
t2g orbital is given as Magnetic moment = n(n  2) B.M
(C) Ligand to metal charge transfer favoured
when the central metal has a high oxidation state Majority of transition metal compounds are
(D) Delocalization of  electrons from the ligand coloured both in solid state as well as in aqueous
solution.due to d-d transition in which unpaired
to the metal reduces the value of 0 electrons from the lower energy d-orbitals are
74. The magnitude of crystal field stabilization transferred to higher energy d-orbitals. The
energy depends on energy of this transition correspond to the
(A) The nature of ligands radiation in visible region. Thus, when white light
(B) The charge on the metals atom falls on such a transition metal compound, some
(C) The charge on the ligand light energy corresponding to a particular colour
is absorbed and one or more electrons are raised
(D) Position of metal in periodic table
from lower energy set of orbitals to those of
higher energy. With the absorption of radiations
COMPREHENSION QUESTIONS corresponding to specific colour from the white
light, a colour known as the complementary
NISHITH Multimedia India (Pvt.) Ltd.,

Passage-I colour is observed or transmitted.


The E 0 values are 78. The compound which have the same
Zn 2  / Zn : 0.76V , Fe 2 / Fe : 0.44V , magnetic moment like that of FeCl2 .

Ni 2  / Ni : 0.25V A) CrCl3 B) MnCl2

Cu 2 / Cu : 0.34V , Ag  / Ag : 0.80V , C) CoCl3 D) NiCl2

Mn2 / Mn : 1.21V 79. A compound of metal ion M x  ( z  24) has a

Pt 2  / Pt :  1.20V spin only magnetic moment of 15 B.M. The


75. The most unreactive metal is number of unpaired electrons in the metal
(A) Zn (B) Fe (C) Ni (D) Pt ion of the compound are
76. The element that does not displace hydro- A) 2 B) 3
gen from dilute acids is C) 4 D) 5
(A) Zn (B) Mn (C) Cu (D) Fe 80. For which one of the following ions, the colour
77. The metal that does not displace Cu from is not due to a d-d transition:
the CuSO4 solution is A) CrO42 B) Cu  NH 3 24
(A) Zn (B) Fe (C) Mg (D) Ag C) Ti H 2O 36 D) CoF63
Passage-II Passage-III
The transition metals and their compounds have A metal complex having composit ion
paramagnetic properties.. This is due to the [Cr(NH3)4Br2I] was isolated in two forms(X)
reason that ions of transition metals have unpaired and (Y). Form (X) reacts with AgNO3 to give a
electrons in (n  1)d orbitals. As the number of pale yellow precipitate which is partially soluble
unpaired electrons increases from one to five in in excess of NH4OH whereas (Y) gives a green-
moving from Sc to Mn, the paramagnetic ish yellow precipitate which is insoluble in
character increases accordingly. From Mn NH4OH.

118 NISHITH Multimedia India (Pvt.) Ltd.,


JEE
JEEADVANCED - VOL
MAINS - CW - VOL- VII
-I d-BLOCK & CO-ORDINATION COMPONDS
RELATIONS
81. 86. Match the following:
(A) The formula of (X) and (Y) are List-I List-II
(Property) (Transition elements)
[Cr(NH3)4 Br I ] Br and [Cr(NH3)3 Br2I] NH3 (A) Highest oxidation state p) Cr
respectively (B) Highest density q) Os
(B) The formula of (X) and (Y) are[Cr(NH3)4Br I (C) Elements with maximum r) Tc
]Br & [Cr(NH3)4Br2]I repectively unpaired electrons
(C) The formula of (X)and (Y) are both (D) Radioactive transition s) Ru
[Cr(NH3)4I]Br2 element
(D) The formula of (X) and (Y) are 87. Match the following
[Cr(NH3)2IBr2](NH3)2 Column - I Column - II
82. Both the (X) form and (Y) form show (molar conductance)
(A)linkage isomerism a) 229 p)  Pt  NH 3 5 Cl  Cl3
(B) Coordination isomerism
(C) Ionization isomerism b) 0 q)  Pt  NH 3 2 Cl 4 
(D) None of these
c) 404 r)  Pt  NH 3  4 Cl 2  Cl 2
83. Which of the following statement is true?
(A) (X) -cis form optically inactive (Y) - cis d) 523 s)  Pt  NH 3 6  Cl 4
form optically active
(B) (X) - cis form optically inactive(Y) -trans t)  Co  N H 3 3 C l 3 
form optically active 88. Match the following
NISHITH Multimedia India (Pvt.) Ltd.,

(C) The cis and trans forms of both X and Y are Column - I Column - II
optically active a) K 2  NiCl4  p) sp3 hybridized
(D) The cis and trans form of both X and Y are
optically inactive. b)  Ni  CO 4  q) para magnatic
3
MATRIX MATCHING QUESTIONS c)  Cr  NH 3 6  r) outer orbital complex
3
84. Match the pair of substances having similar d)  FeF6  s) digmagnatic
properties 89. Match the following
Column – I Column - I Column - II
4
a) NiSO4 and VO+ a)  Fe  CN 6  p) Paramagnetic, sp3d 2
b) TiCl4 and ZnSO4 2
b)  Fe  H 2O 6  q) Diamagnetic, d 2sp3
c) MnCl3(aq) and CoCl3(aq)
d) FeCl3 and MnSO4 c)  CrCl2  NH3 4  NO3 r) Diamagnetic sp3d 2
Column – II 2
p) Same magnetic moment d)  Zn  H 2O 6  s) Ionization isomerism
q) Nearly similar colour 90. Match the following
r) Same oxidation state Column - I Column - II
2
s) Same outer electronic configuration a)  Ni  NH3 6  p) Diagmagnetic
85. Match the following:
List-I List-II b)  Ni  CO 4  q) sp 3 d 2
Metals Characteristic
3
(A) Cd (p) d-block metal c)  Cr  H 2O 6  r) d 2sp3
(B) Rh (q) Transition metal
2
(C) Fm (r) Inner transition metal d)  Ni  CN 4  s) Inner orbital
(D) Gd (s) Lanthanide
(t) Actinide complex
NISHITH Multimedia India (Pvt.) Ltd., 119
d-BLOCK & CO-ORDINATION COMPONDS JEE ADVANCED - VOL - VII
91. Match the following 95. Match the following
Column - I Column - II Column -I Column-II
A) [Co(NH3)4Cl2] p) Optical isomerism
a)  Cu  NH3 4  H 2O 2  Cl2 p) Geometrical B) [Co(en)3]Cl2 q) Ionization
isomers isomerism
C) [Co(en)2(NO2)Cl]SCN r) Coordination
b)  Pt  NH3 2 Cl2  q) Diamagnetic isomerism
D) [Co(NH3)6][Cr(CN)6] s) Geometrical
c)  Co  H 2O 5 Cl  Cl r) Paramagnetic isomerism

d)  Cr  H 2O 6  Cl3 s) optical isomers


92. Match the following: STATEMENT QUESTIONS
List-I List-II
(A)Statement – 1 is True, Statement – 2 is
3
(A)  CoF6  (p) High spin complex True; Statement – 2 is a correct explanation
for Statement – 1.
3
(B)  Fe  H 2O 6  (q) Low spin complex (B)Statement –1 is True,Statement – 2 is
True; Statement – 2 is NOTa correct
3 explanation for Statement- 1.
(C) Co  en 3  (r) Outer orbital complex
NISHITH Multimedia India (Pvt.) Ltd.,

(C)Statement – 1 is True, Statement – 2 is


False.
(D) Cr  CO6  (s) Inner orbital complex (D)Statement – 1 is False, Statement – 2 is
(t) Paramagnetic True.
93. Match the following 96. Statement-1: Co  CO  4 is not stable but
List-I List-II 
Co  CO  4  is stable.
(A)  Pt  NH 3 2 Cl2  (p) Dipole Moment

Statement-2: Co  CO 4  obeys EAN rule
(B) K3 Cr  OX 3  (q) Polarimeter
97. Statement-1: W  CO 6  obeys EAN rule as it
(C) Co  NH 3  4 Cl2  Cl (r) Precipitation with attains Xe configuration.
Statement-2: It is stable because it attains
AgNO3
Rn configuration.
(D) Co  en 2 Cl2  (s) Cation exchange 98. Statement-1: The [Ni(en)3]Cl2 has lower sta-
Resin bility than [Ni(NH3)6 ] Cl2 because
(t) Anion exchange Statement-2: In [Ni(en)3]Cl2, the geometry of
Ni is octahedral.
Resin
94. Match the following
List-I List-II
INTEGER TYPE QUESTIONS
(A) Wilkinson catalyst (p) +1 O.S.
(B) deoxy haemoglobin (q) +2 O.S. 99. The oxide formed in the maximum oxidation
(C) Brown Ring complex (r) Coordination state is Zr Ox . x is
number 4
100. How many of the following cannot displace
(D) Vaskas complex (s)Coordination
number 6 H 2 from dil.HCl ?
(t) Neutral complex Cu, Zn, Ni, Fe, Ag , Au, Pt , Hg , Cd

120 NISHITH Multimedia India (Pvt.) Ltd.,


JEE
JEEADVANCED - VOL
MAINS - CW - VOL- VII
-I d-BLOCK & CO-ORDINATION COMPONDS
RELATIONS
101. How many of the following reactions cannot 113. A compound of vanadium has a magnetic mo-
occur ?
ment    of 1.73BM. If the vanadium ion
(1) Fe  2 HCl  FeCl2  H 2 
in the compound is present as Vx+ , then, the
(2) Cu  2 HCl  CuCl2  H 2 
value of x is ? __
(3) Cu  ZnSO4  CuSO4  Zn  114. Number of outer most ‘d’ electrons in
Rhodium are
(4) Zn  2 AgNO3  Zn  NO3 2  2 Ag 
115. The sum of ns and (n-1)d electrons in Tc
(5) Cu  2 AgNO3  Cu  NO3  2  2 Ag  are
116. How many statements given below are
(6) Fe  CuSO4  FeSO4  Cu  correct ?
102. The number of transition metals in the alloy
i) Pyrolusite on fusion with KNO2 in NaOH gives
Alnico is
2 K 2 MnO4
103. The secondary valency in Co  EDTA  
is ii) Potassium manganate  K 2 MnO4  on
104. How many of the following are diamagnetic electrolytic oxidation gives KMnO4
?
iii) Potassium manganate  K 2 MnO4  is formed
NISHITH Multimedia India (Pvt.) Ltd.,

1 2
 Ag  NH 3  2  , Cd  NH 3 4  ,
, when formaldehyde reacts with potassium
2 2
Cr  CO 6 ,  Ni  NH 3 6  ,  Ni  CN  4  permanganate in acidic medium
iv) Potassium manganate  K 2 MnO4  is formed
105. The number of isomers possible for
, when potassium permanganate is moderately
 Pt  Cl  Br  NH 3  py   is
heated with potassium hydroxide.
106. The number of oxygen atoms involved in 117. Number of correct statements among the
bonding in the coordination sphere of following are
2 i) Ferrous sulphate is used for making blue black
 Mg  EDTA  is
ink.
107. How many number of unpaired electrons ii) Ferrous sulphate is a strong reducing agent
present in [Cr(en)3]+2 ? iii) Hydrated ferrous sulphate is a green crystalline
108. How many geometrical isomers are possible
compound
for [Pt (NO2) (NH3) (NH2OH) (Py)]+?
iv) The aqueous ferrous sulphate is slightly acidic
109. How many stereoisomers are possible for
due to hydrolysis
[Pt (Br) (Cl)  NH 3  2 ] ? 118. How many of the following are correct about
110. Dimethyl glyoxime forms a square planar the stability of oxidation states of
complex with Ni+2. This complex contain how manganese?
many number of unpaired electrons? i) Mn  II   Mn VII   (acidic solution)
111. How many number of d-electrons are present
in [Co (NH3)5 CO3]ClO4 ? ii) Mn  II   Mn  IV   (alkaline solution)
x+
112. A compound of metal ion M (Z = 24) has a
iii) Mn VI   Mn  IV   (acidic solution)
spin only magnetic moment of 15 B.M. The
number of unpaired electrons in the iv) Mn VI   Mn VII   (strong alkaline

compound are solution)

NISHITH Multimedia India (Pvt.) Ltd., 121


d-BLOCK & CO-ORDINATION COMPONDS JEE ADVANCED - VOL - VII

LEVEL-V 94)A - p,r,t ; B - q,s,t ; C - p, s; D - p, r, t


KEY 95)A - s ; B - p ; C - p,q,s ; D - r
SINGLE ANSWER
01) A 02) D 03) D 04) D 05) B STATEMENT TYPE
06) C 07) C 08) B 09) C 10) B 96) A 97) D 98) D
11) C 12) C 13) B 14) C 15) C
16) A 17) B 18) A 19) A 20) A INTEGER TYPE
21) A 22) B 23) D 24) A 25) C 99) 2 100) 5 101) 2 102) 2
26) D 27) C 28) B 29) D 30) B 103) 6 104) 4 105) 3 106) 4 107) 2
31) D 32) A 33) C 34) A 35) B 108)3 109) 2 110) 0 111) 6 112) 3
36) A 37) A 38) B 39) C 40) B 113) 4 114) 8 115) 7 116) 3 117) 4
41) D 42) B 43) D 44) D 118) 3

MULTIPLE ANSWER LEVEL -V


45) A,D 46) A,C 47) A,C 48) B,D
49) A,B,C 50) B,C,D 51) C,D HINTS
52)A,B,C 53) A,B,C,D 54) A,B,D
NISHITH Multimedia India (Pvt.) Ltd.,

55) A,B 56) A,B,D 57)A,C,D 58) B,C,D 2. W belongs to VI B group. It has high MP.
59) A,D 60) B,C 61) A,B 62) A,B,D It has greater metal - metal bond strength.
63) A,B,C,D 64) A,B,C 65) A,B,C,D 3
66) A,D 67) A,B,C,D 68) B,C,D 4. FCC lattice. One plane slides over another. ef-
69) B,C,D 70) B,C,D 71) B, D fective M - M bond strength
72) C 73) A,D 74) A,B,D
5. Zn,3d 10 , 4 s 2 . Weak metal-metal bond. Puri-
COMPREHENSION TYPE fied by distillation.
Passage-I 20. F  is a weak field ligand.
75) D 76) C 77) D 21. en is a strong field ligand.
Passage-II 22. Ti 3 : 3d 1
78)C 79) B 80)A
Passage-III 23. dsp 3 , coordination number : 5
81) B 82) C 83) D 1 1
24. KD   3
 5 102
K F 2 10
MATRIX MATCHING TYPE
84) a - p, q; b - p, q; c - p, q, r; d - p, s 25. H 2O is ‘aqua’ and not ‘aquo’
85)A: p ;B: p,q ; C: r,t ; D: r,s 26. Organic amine and for NH 3 - ammine. Note
86) A-q,s B-q, C-p D-r the spelling
87) A-r B- q,t C-p , D- s
27. They have d 0 configuration.
88) A-p,q B- p,s C-q , D- q,r
89) A-q, B- p, C-s, D- r 28. d 6 , 4 unpaired electrons, highspin complex.
90) A-q, B- p, C-r,s, D- p 29. F  is a weak field ligand, NH 3 is strong field
91)A-p,r, B- p,q , C-r , D- r
ligand.
92) A-p,r,t, B - p,r,t C - q,s, D - q, s
30. It has both  and  bonding
93) A - p ; B - q,t ; C - p,r,s ; D - p,q
122 NISHITH Multimedia India (Pvt.) Ltd.,
JEE
JEEADVANCED - VOL
MAINS - CW - VOL- VII
-I d-BLOCK & CO-ORDINATION COMPONDS
RELATIONS
31. 1
43. In HN 3 the O.S. of nitrogen is  , and
. 1 3
hydrogen +1.
44. 1) In hydrogen atom 3s,3 p ,3d has same
energy
2) dx 2  y 2 the electron density is along
32. SOL : Zn  H 2 SO 4  ZnSO 4  H 2 xy plane
Zn  conc.2 HCl  ZnCl2  H 2 3) Cr is a d-block element, the differentiating
Zn  4 HNO3  Zn  NO3  2  2 NO2  2 H 2O electron enters  n  1 d orbital.
33 : [M(aa)2b2] Type. Cis isomer occurs in 4) the three Quantum Numbers n, l, m are
two enantiomeric forms. given by schrodinger wave equation.
34: [Cr(H2O)6]3+ Cr3+ - 4s03d3
4+
[Mn(H2O)6] Mn4+ - 4s03d3 MULTIPLE ANSWER
Same no.of unpaired electrons.

35.  NH 4  2 Cr2O7   Cr2O3  N 2  4 H 2O 45. Mn7 and Cr 6 can oxidize I  to I 2 .
Hence green coloured powder blown in the air
NISHITH Multimedia India (Pvt.) Ltd.,

0
46. Cu cannot displace as it has higher ESRP (+ve)
is Cr2 O3
36. Misch metal is 40.5% Ce, 44% La and Nd, value.
5% Fe and others. 47. TiCl4 is covalent. Metalic bond strength is weak
37. CrO42 is colour due to charge transfer.. in Hg.
48. Highest oxidation state. Oxides are acidic
38. Au 3 C.N. 4, Pt 4 C.N. 6 49. Transition metal compounds that are covalent
39. G of AgO is very close to positive value, undergo hydrolysis. CaCl2 is ionic and no hy-
therefore thermally less stable.
2
drolysis.
40. Cr 6 in basic medium exist as CrO4 , 50. Resistant to oxidation.
colour yellow. 53. All are diamagnetic.
41. H 2O2  H 2O  O  59. Water of hydration exists in ionic com-
K 2Cr2O7  H 2 SO4  H 2Cr2O7  K 2 SO4 pounds. d x2  y 2 orbitals is used in dsp 2 hybrid-
H2Cr2O7  4O  2CrO5  H2O ization.

4CrO5  6H2 SO4  2Cr2  SO4 3  6H2O  7O2 60. Ni 2  d 8  or Cr 3  d 3  form octahedral com-
plexes whether the ligand is strong or weak field
CrO5 is blue in colour.. ligand, they have same magnetic moment. They
42. Fe3  3d 5 are all colored.
61. A & B : 3d 1 configuration.

5 unpaired electrons 62.  NiCl4 2 is tetrahedral


Co 2  3d 7 63. (A): Ni  O  :3d 10 ; (B): Fe3 : 3d 5
(C) Mn 2  : 3d 5 ; (D) Co 2  : 3d 7 . All have sp 3
3 unpaired electrons
hybridization.
NISHITH Multimedia India (Pvt.) Ltd., 123
d-BLOCK & CO-ORDINATION COMPONDS JEE ADVANCED - VOL - VII
64. D is paramagnetic (2 unpaired e ). 79. X and Y are actually ionisation isomers
65. All are strong field ligands. Hence, Choice (C) is correct, while (A) and (B)
66. B and D are colored due to charge transfer tran- are incorrect.
sition. 80. [Cr(NH3)4IBr]+ and [Cr(NH3)4Br2]+ are Ma4 bc
67. small ligands, can act as bridging ligands. type and Ma4b2 type respectively.
68. They are ambidentate ligands. Both show geometrical isomerism but each
69. In ‘A’, for any position of b gives identical iso- isomer is optically inactive.
mer. Hence a , b and c are incorrect.
70. A is a trans isomer. Others are cis isomers.
MATRIX MATCHING TYPE
71. (1) and (3) are symmetrical not opticaly acitve.
 5 has Ti 4  d 0  configuration. 94. Vaskas complex is Trans-

72. CN   F (Field higher the charge on the metal


 IrCl  CO  PPh 3 2 
greater the stability. 6 coordinate complexes are
more stable than four coordinated one. STATEMENT TYPE
73. (A) is correct answer
(B) is incorrect because  donor ligand forms

 bond and C is incorrect.
NISHITH Multimedia India (Pvt.) Ltd.,

96. Co  CO  4  : 27  4  2  1  36e 


(D) is correct answer.
74. (A) is correct because strong field ligand cause 97. W  CO 6  :74  6  2  86  Rn 
more splitting
98. The chelates are more stable. Statement-1 is
(B) is correct because 0 increases with increas-
false.
ing oxidation number of metal.
(C) is incorrect because 0 is independent of INTEGER TYPE
charge on ligand.
(D) is correct because 0 increases down the 99. ZrO2
group. 0
100. They have ESRP   ve , Cu, Ag , Au, Pt , Hg
COMPREHENSION TYPE 101. 2 and 3. Refer the electro chemical series.
102. Ni and Co.is two
Passage-I
75. High positive E 0SRP value. 103. EDTA : Tetravalent hexadentate ligand is 6
2
76. Positive E 0SRP value. 104.  Ni  NH 3 6  is paramagnetic.
77. More positive E 0SRP value. 105. Square planar complex
Passage-II 106. The four tetraacetate oxygen atoms are involved
78. Since ‘X’ gives pale yellow ppt and ‘Y’ gives in coordination.
yellow ppt. Hence, in ‘X’ Br is ionisable while in 2 4
‘Y’ I is ionisable. 107. Cr :3d :         
X = [Cr(NH3)4Br I ]Br 110. Ni 2 : 3d 8 , dsp 2 ,        
Y = [Cr(NH3)4Br2]I
Hence choice (B) is correct while a, c, and d are 111. x  0  2  1  0, x  3, Co3 : 3d 6
ruled out.
124 NISHITH Multimedia India (Pvt.) Ltd.,
JEE
JEEADVANCED - VOL
MAINS - CW - VOL- VII
-I d-BLOCK & CO-ORDINATION COMPONDS
RELATIONS
6. The metal that has the least melting point
112.   n  n  2  , where n  no. of unpaired among the following is
electrons. (A) Mn (B) Fe (C) Cr D)W
7. The metal that forms interstitial nitride is
113. Vanadium .E.C  3d 3 4 s 2
(A) Mg (B) Ca (C) Cr (D) Li
and +4 O.S. V 4  3d 1 8. Which of the following is paramagnetic

114. Rh E.C  4 d 8 5s1 A) V  Co 6  B) Ir4  Co 12

115. Tc E.C  4d 6 5s1 C) Cr  Co 6  D) Fe2  Co 9


116. Conceptual 9 The correct statement about iron includes
117. Conceptual
I) Fe3O 4 is a mixed oxide of iron
118. Mn in acid media changes from +7 to +2, in
neutral media +7 to +4, in strong alkaline media II) that the iron shows +2 oxidation state with
+7 to +6. six electrons in the unfilled 3d orbitals
(III) the common oxidation state iron is +3 with
LEVEL - VI five unpaired electrons in the 3d orbitals
A)I,II,III B) I,II
C) II,III D) I only
NISHITH Multimedia India (Pvt.) Ltd.,

SINGLE ANSWER QUESTIONS


10. Which is highly colored due to inter valence
1. Which one of the forms a metallide M    electron transfer transition ?
easily ? (A) FeO (B) Cr2 O3
(A) Na (B) Au (C)Ti (D) Zn (C) Fe3O4 (D) CuO
2. The pair that can exhibit more stable +4 oxi-
11 The transition metals exhibit higher enthal-
dation state is
pies of atomisation due to
(A) Fe, Ni (B) Ag, Au
A) their ability to show varible oxidation states
(C) Pt, Pd (D) Cd, Hg
B) the presence of incompletely filled
3. Which of the following electronic
d-subshell
configurations is associated with the high-
C) their ability to exist in the solid state with un-
est stable oxidation state
paired electrons
A)  Ar  3d1 4s 2 B)  Ar  3d 5 4s1 D) Strongly interatomic interaction aries
because of having large number of unpaired
C)  Ar  3d 5 4s 2 D)  Ar  3d 6 4s 2 electrons in their atoms
4. The alloy that is used for making permanent 12 Which of the following statement are cor-
magnet is rect about Zn, Cd and Hg ?
(A) Al, Ni, Co (B) Pb, Sn, Bi I) they exibit high enthalpies of atomisation as
the d-subshell is full
(C) Fe, Mn, Cu (D) Cu, Au, Hg II) Zn and Cd do not show varible oxidation states
5. Which one of the following pairs is colored while Hg shows +1 and +II
? III) Compound of Zn, Cd anf Hg are
(A) TiCl3 ,VCl2 (B) TiCl4 , Cu2Cl2 paramagnetic in nature
IV) Zn, Cd and Hg are called soft metals
(C) Hg 2Cl2 , anhydrous CuSO4 A) I,II,III B) I,II
(D) AgNO3 , Cd  NO3  2 C) II,IV D) IV only
NISHITH Multimedia India (Pvt.) Ltd., 125
d-BLOCK & CO-ORDINATION COMPONDS JEE ADVANCED - VOL - VII
13 In the calculation of magnetic moment, the 18 A compound of metal ion M x   z  25  has
orbital magnetic moment contribution is neg-
ligible for the following ion a spin only magnetic moment of 15 B.M .
A) Pt 2 B) Mo  2 The positive O.S. of the metal is
A) 2 B) 3
C) Pd2 (D) Ti 3 C) 4 D) 5
19 Amongst
14 The acidic character of manganese is cor-
rect TiF62 .CoF63 , Cu 2Cl2 and NiCl 42 , t h e
A) MnO  Mn 2O3  MnO 2  Mn 2O 7 colurless species are
A) CoF63 and NiCl24 
B) MnO  MnO 2  Mn 2 O 3  Mn 2 O 7
B) TiF62  and CoF63
C) MnO  Mn 2O3  MnO 2  Mn 2O 7
C) TiF62  and Cu 2Cl 2
D) Mn 2O 7  Mn 2O3  MnO 2  MnO
D) NiCl24  and Cu 2Cl 2
15 Among the following series of transition
metal ions, the one where all metal ions have 20 The magnetic moment of two ions

3d 2 electronic configuration is M x  and M y of the element M(Z=26) is


NISHITH Multimedia India (Pvt.) Ltd.,

found to be 5.916B.M . If x>y, then which of


A) Ti 3 , V 2 ,Cr 3 , Mn 4 the following statement is correct?
A) M y is more stable than M x 
B) Ti  ,V 4 ,Cr 6 , Mn 5
B) M y is less stable than M x 
C) Ti 4 , V 3 ,Cr 2 ,Mn 3 C) Both are equally stable
D) Can not be predicated
D) Ti 2 , V 3 ,Cr 4 ,Mn 5 21 The magnetic moment of ion is close to
16 Which of the following is not responsible for 36 10 24 joule/Tesla. The number of un-
the catalytical activity of transition metals paired electrons of the ion are
and their compounds? A)4 B) 2 C) 1 D) 3
22 Which of the following compounds are
A) Transition metals have large surface area
coloured substances ?
B) Transition metals show varible oxidation states
C) Transition metals form intermediate complexes I  Ag 2CO 3 II  Ag 3PO 4
D) Transition metals are coloured III  Agl IV  Ag 2S
17 The melting points and the boiling points A) Only III
of the transition elements are higher than B) only III and IV
the corresponding s-block elements. This is C) only I,III and IV
because D) all the four
A) Transition metals have smaller size 23. In Zieses salt, the C = C bond length is
B) Of the presence of one or more unpaired elec- C  C bond length in ethane is1.54A0 
trons contribution to higher inter atomic forces  
on account of covalent bond Note C  C bond length in ethene is1.34A0 
 0
C) of strong metalic bond due to small size and C  C bond length in ethyne is1.20A 
higher ionization energy
(A) 1.37 A0 (B) 1.19 A0
D) of the presence of vacant d-orbitals
(C) 1.87 A0 (D) 1.34 A0
126 NISHITH Multimedia India (Pvt.) Ltd.,
JEE
JEEADVANCED - VOL
MAINS - CW - VOL- VII
-I d-BLOCK & CO-ORDINATION COMPONDS
RELATIONS
24. Which one of the following can get oxidized 31. The IUPAC name of ferrocene is
easily ? (A) bis (  5 - cyclopentadienylo) iron (O)
3 2
(A) Co  NH 3 6  (B) Co  NH 3 6  (B) bis ( 10 -cyclopentadienylo)iron (O)

(C) Fe  CO 5 (D) Cr  CO 6 (C) bis ( 5 - cyclopentadienylo)iron (II)

25. The magnetic moments (BM) of the com- (D) bis (  10 - cyclopentadienylo)iron (II)
2 2 32. Cis di -  - chloro bis [chloro(triphenyl phos-
plexes  Ni  H 2 O 6  and  Ni  NH 3 6  phine) platinum (II)] is
are resectively Cl Cl Pph3
(A) 1.73, 2.84 (B) 0, 2.84 Pt Pt
(A)
(C) 2.84, 0 (D) 2.84, 2.84 Ph3 P Cl Cl
26. The number of electrons in t2g orbitals in .

K 4  Fe  CN 6  is Cl Cl Cl
Pt Pt
(A) 2 (B) 3 (C) 6 (D) 5 (B)
2
Ph3 P Cl Pph3
27. The CFSE of  Mn  H 2 O 6  is
NISHITH Multimedia India (Pvt.) Ltd.,

(A) 1.2 (B) 1.9  Cl Cl


P h3 P
Pt Pt
(C) 0 (D) 2.4  (C)
P h3 P Cl Cl
28. The theoretical percentage weight loss when
Cr  H 2 O  4 Cl2  Cl .2 H 2O is kept over Cl
Pt Cl Cl

conc. H 2 SO4 in a closed desicator is Pt


(D) Ph3 P
(A) 13.50% (B) 6.75% (C) 18% (D) 36% Ph3 P
29. 0.2435g of a complex gave 0.2870g of AgCl 33. Which one of the following represents a Cis
when treated with a excess AgNO3 solution. isomer ?
The complex is a a
(A) Cr  NH 3  4 Cl2  Cl a a b a

(B) Cr  NH 3 5 Cl  Cl2 M M


(A) (B)
(C) Cr  NH 3 3 Cl3  b b b b
b b
(D) Cr  NH 3 6  Cl3

30.
3
 Fe  CN 5  CO   IUPAC name is a a
b b a b
(A) pentacyanocarbonylferrate (II)
(B) carbonylpentacyanidoferrate (II) M M
(C) pentacyanocarbonyliron(III) (C) (D)
(D) carbonylpentacyanoiron (III) b b b a
a a
NISHITH Multimedia India (Pvt.) Ltd., 127
d-BLOCK & CO-ORDINATION COMPONDS JEE ADVANCED - VOL - VII
34. The total number of possible geometrical iso- 39. The neutral and positive charged species
mers in Mabcdef where a to f are that can coordinate to a metal atom is
different monodentate ligands is given by (A) Cl  (B) NH 3 (C) CO (D) NO
(A) 4C2 (B) 5C1 (C) 3C2 (D) 6C2 40. Which of the following pair of compounds is
35. The complex formed by Pt with 2 expected to exhibit same colour in aqueous
solution?
diethylenetriamine (dien) and chlorido ligand
is given by (A) FeCl2, CuCl2 (B) VOCl2, CuCl2
(C) VOCl2, FeCl2 (D) FeCl2, MnCl2
2
(A)  Pt  dien   41. The oxidation state of Mo in its oxido-com-
2
(B)  P t  d ien  C l 2 
2
plex species Mo2O4  C2H4 2 H2O 2  is
1 (A) +2 (B) +3 (C) +4 (D) +5
(C)  Pt  dien  Cl  4-
42. The donor sites of (EDTA) are
2
(D)  Pt  dien  C l  (A) O atoms only
36. Which one of the following give an acid when (B) N atoms only
passed through cation exchange (C) Two N atoms and four O atoms
(D) Three N atoms and three O atoms
resin  RSO3 H  ?
NISHITH Multimedia India (Pvt.) Ltd.,

43. In nitroprusside ion, the iron and NO exist


(A) K 4  Fe  CN 6  as Fe2+ and NO+ rather than Fe3+ and NO.
These forms can be differentiated by
(B) K 3  Al  C2 O4 3  (A) Estimating the concentration of iron
(B) Measuring the concentration of CN–
(C)  Co  NH 3 6  Cl3
(C) Measuring the solid state magnetic
(D) Cr  Co 6  moment
(D) Thermally decomposing the compound
37. Which one of the following give a base when
44. In the isoelectronic series of metal carbo-
passed through anion exchange
nyl, the CO bond strength is expected to
resin  R4 N   OH  ? incease in the order
 
(A) K 4  Fe  CN 6  (B) Co  en 3  Cl3 (A) Mn  CO 6   Cr  CO 6    V(CO)6 
 
(C) Co  NH 3  6  Cl3 (D)  Fe  H 2 O 6  SO4 (B)  V(CO)6   Cr  CO 6   Mn  CO6 
38. Which one of the following does not form a  
chelate with oxalate ion by reacting in aque- (C)  V(CO)6   Mn  CO 6   Cr  CO 6 
ous solution ?  
(D)  Cr  CO 6   Mn  CO 6    V(CO)6 
1
(A) cis  Co  NH 3  4 Br2  45. The magnetic moments of complexes given
 below are in the order
(B) cis  Co  NH3  4 Cl2 
4

1
I) [Ni(CO)4] II)  Mn  CN 6 
(C) cis  Co  en  2 Cl2 
III)[Cr(NH3)6]3+ IV) [CoF6]3-
(D) trans  Co  en 2 Cl2 
1 (A) I > II > III > IV (B) I < II < III < IV
(C) IV > II > I > III (D) IV < II < I < III
128 NISHITH Multimedia India (Pvt.) Ltd.,
JEE
JEEADVANCED - VOL
MAINS - CW - VOL- VII
-I d-BLOCK & CO-ORDINATION COMPONDS
RELATIONS
46. Three arrangement are shown for the 50. Which of the following can show geometri-
 cal isomerism?
complex  CoBr2  NH 3  2  en   . Which one
(A) [Pt(NH2  CH  CH  NH2 )2 ]2 
is wrong statement? | |
CH3 CH3
(B) [Co(en)2(NH3)Cl]2+
(C) Both a and b
(D) None of these
51. The complex which does not obey EAN rule
is
0
(A)  Fe  CO5 
0
(B) Cr  CO 3  NO 2 
0
(C)  Fe  CO 3  NO 2 
(A) I and II are geometrical isomer 0
(B) II and III are optical isomers (D)  Fe  CO 2  NO 2 
(C) I and III are optical isomers
NISHITH Multimedia India (Pvt.) Ltd.,

(D) II and III are geometrical isomers MULTIPLE ANSWER QUESTIONS


47. Among the following , which is not the  -
bonded organometallic compound ? 52 The correct stability order of the oxidation
states is
(A)  CH3 4 Sn
(A) Mn2  Fe2 (B) Ni 3  Fe3
6
(B) Cr   C6H6  2
(C) Fe3  Ni 3 (D) Co 2  Ni 2 
53. The compounds that cannot exist
 5
(C) Fe   C5H5  2
(A) WO4  (B) NbCl6
(C) HfCl4 (D) ZrO2
(D) K PtCl3    C2H4 
2
54. Which of the following oxides are amphot-
48. Complex compounds(s) of which optical ac- eric ?
tivity does not depend upon the orientation (A) V 5 (B) Zn 2 (C) Fe 2 (D) Fe3
of the ligands around metal cation 55. Which of the following are nonferrous alloys
i) [CoCl3(NH3)3] ?
ii) [Co(en)3]Cl3 (A) Brass (B) Bronze
(C) Chromium Steel (D) German Silver
iii) [Co(C2O4)2(NH3)2]-
56. In the formation of interstitial compounds
iv) [CrCl2(NH3)2(en)]+ by transition metals, identify the correct
(A) II, III and Iv (B) I, II and IV statements
(C) II and IV (D) Only II (A) Melting points of interstial carbides are more
49. The total possible coordination isomers for than those of respective pure metals
the following compounds respectively are (B) Densities of interstial hydrides are less than
[Co(en)3][Cr(C2O4)3] those respective pure metals
(C)Electrical conductivity is lost in the formation
[Cu(NH3)4[CuCl4] of interstial compound from a metal
[Ni(en)3][Co(NO2)6] (D) Interstitial borides are very hard in nature
(A) 4,4,4 (B) 2,2,2 (C) 2,2,4 (D) 4,2,4 and less reactive than the parent metals
NISHITH Multimedia India (Pvt.) Ltd., 129
d-BLOCK & CO-ORDINATION COMPONDS JEE ADVANCED - VOL - VII
4 K in liq
57.
4
Complex ions  NiCl6  ,  Ni  CN 6  61. K 2  Ni  CN 4  
NH
 'X ' regarding
3

similar in their given properties : this reaction correct statement is/are


(A) oxidation state, geometry
(A) ‘X’ is K 4  Ni  CN 4 
(B)co-ordination number , EAN
(C)Magnetic moment,geometry (B) The oxidation state of Ni changed +2
(D)stability, colour to zero
58. Select correct statement(s) regarding given (C) The structure of ‘X’ is tetrahedral
complexes 2
(D)  Ni  CN 4  is square planar
(A)  Fe  CO 5  , the orbitals used for
complex
hybridization in Fe atom are s, 62. Which one of the following statement(s)
p x ,p y , pz ,d is/ are false
z2
(A) Weak ligands like F , Cl and OH 
(B)  Pt  NH3 2 Cl2  , the orbitlas used
usually form low spin complexes
for hybrization on Pt atom are s,
px ,p y ,d (B) Strong ligand like CN  and NO 2 ,
x 2  y2 and it is low spin complex
generally form high spin complexes
NISHITH Multimedia India (Pvt.) Ltd.,

3
(C)  Cr  H 2O 6  , the orbitals used for (C)  FeF6 
3
is high spin complex
hybridization in Cr atom are s
(D)  Ni  CO 4  is high spin complex
px , p y ,p z ,d 2 ,d and it is high spin
z x 2  y2
63. Which of the following is correct about
complex
tetrraammine dithiocyanato-s cobalt (III)
(D) Ni  CO 4 , the orbitals used for tris (oxalato) cobaltate(III)
hybrization in Ni atom are s A) Formula of the complex is
p x , p y ,p z and it is low spin complex  Co  SCN 2  NH 3 4  Co  ox 3 
59. Complex compound B) It is a chelating complex and show
linkage isomerism
 Co  SCN 2  NH 3 4  Cl exhibit C) It shows optical isomerism
(A) ionization isomerism D) It shows geometrical isomerism
64. Select the correct statement :
(B) Geometrical isomerism
A) Chelation effect is maximum for five
(C) optical isomerism and six membered rings
(D) linkage isomerism B) Greater the charge on the central metal
60. Which of the following compound has/ cation, greater , the value of  (CFSE)
have effective atomic number equal to the 3
atomic numer of a noble gas C) In complex ion  CoF6  , F is a weak

filed ligand so that  oct  p (Pairing


A) K Co  CO 4  (B) K 2 [Fe  CO  4 ]
energy)and it is low spin complex

C)  Co  NH3 6  Cl2 (D)  CoCl3  H 2 O 3  D) [CoCl2  NH3 2  en ] complex ion
will have four different isomers
130 NISHITH Multimedia India (Pvt.) Ltd.,
JEE
JEEADVANCED - VOL
MAINS - CW - VOL- VII
-I d-BLOCK & CO-ORDINATION COMPONDS
RELATIONS
65. Which of the following obey 18e  rule ? 72. Which is/are correct statement (s)?
(A) [Ag(NH3)2]+is linear with sp hybridsed Ag+
(A) Cr  CO 6 (B) Ni  CO 4
ions
(C) Cr  C6 H 6  2 (D) Fe  C5 H 5  2 (B) NiCl42- , CrO42- and MnO4– have tetrahe-
66. Which of the following obey EAN rule ? dral geometry
3
(C) [Cu(NH 3 ) 4] 2+ , [Pt(NH 3 ) 4 ] 2+ and
(A) Co  NH 3 6  (B) Cr  CO 6 [Ni(CN)4]2- have dsp2 hybridisation of the metal
 
ion
(C) V  CO 6  (D) Co  CO 4  (D) Fe(CO)5 has trigonal bipyramidal structure
67. Which of the following are four coordinated with dz2 sp3 hybridised ion
complexes? 73. Which of the following is/are correct
(A) Be  acac 2 relations(s)?
2 4 4
(B)  Zn  EDTA   (A) 0  t (B)  t  0
9 9

(C)  P t  d ien  C l  (dien = diethylene triamine) 3
(C)  s  1.30 (D)  t  0
2 7
(D)  Pt  trien   (trien = t riethylene 74. Which of the following is /are correct about
NISHITH Multimedia India (Pvt.) Ltd.,

tetraamine) Wilkinson’s catalyst?


68. In Ni 2  octahedral complexes, which of the (A) It is used as homogeneous catalyst for se-
lective hydrogenation of organic molecule at
following ligands do not pair up the electrons
? room temperatuer and pressure
(B) It is tetrahedral complex
(A) NH 3 (B) H 2O (C) en (D) dien (C) It does not have unpaired electrons
69. The  - complexes are (D) Its formula is TiCl4 + Al(C2H5)3
(A) Ferrocene
(B) Zeises salt
COMPREHENSION QUESTIONS
(C) dibenzenechromium(0)
(D) Grignard Reagent
70. Which of the following exhibits optical ac- Passage-I
tivity ? I. (i)
Acetone
 K 3 Co  CN 6   Cr  NH 3 6  Cl3   x  3KCl 
(A) Cis - Co  en  2 Cl2 
(ii)

(B) trans - Co  en  2 Cl2  K 3 Cr  CN  6   Co  NH 3 6  Cl3
Acetone
(C) K 3  Al  C2 O4 3    y  3KCl 
 The solution was filtered.
(D) Co  NH 3  4 Cl2 
x and y are obtained after evoporation of the
71. The optically active complexes are solvent from the filtrate.
(A)  As  CH 3  Cl  Br  I  
 75. x and y are
(A) Linkage isomers
(B)  Pt  NH 3  Py  Cl  Br   (B) Geometrical Isomers
(C) Coordinate Isomers
(C)  Pt  gly  2  (D) Cr  en 3  Cl3 (D) Positional Isomers.
NISHITH Multimedia India (Pvt.) Ltd., 131
d-BLOCK & CO-ORDINATION COMPONDS JEE ADVANCED - VOL - VII
76. When x is sent through cation exchange in in
resin, the complex coming out in solution is 78.  Ki and
i 1

i 1
log K i are respectively
3 3
(A) Co  NH 3 6  (B) Cr  NH 3 6  (A)  n and log  n
3 3
(C) Co  CN 6  (D) Cr  CN 6  (B) K n and log K n
77. When y is sent through anion exchange (C)  n1 and log  n1
resin, then the complex eluted is (D) K n1 and log K n1
3
(A) Co  NH 3 6  79. Choose the correct order
3 (A) K 3  K 2  K1 (B) K 3  K 2  K1
(B) Cr  NH 3 6 
(C) K1  K 2  K 3 (D) K1  K 2  K 3
3
(C) Co  CN 3 6  2
Cd  H 2 O 4   4 NH 3 
3 80. (1) 2
(D) Cr  CN 6  Cd  NH 3 4   4 H 2 O , log 1
Passage-II 2
Consider the following complex  Cd  H 2 O  4   2 en 
equilibrium (2) 2
 Cd  en 2   4 H 2 O , log  2
NISHITH Multimedia India (Pvt.) Ltd.,

M  L  ML , K 1 
 ML 
log  2  log 1 , is due to
 M  L  (A) Enthalpy change

ML  L  ML2 , K 2 
 ML2  (B) Entropy change
(C) Ring strain in the chelate
 ML  L 
(D) Large change in  0
ML2  L  ML3 , K 3 
 ML3 
 ML2  L  Passage-III
Bonding in metal carbonyls is represented by the
MLn 1  L  MLn , K n 
 MLn  end structures.
 MLn 1   L 
M-C O 
 M=C=O
K1 , K 2 , K 3 are step wise stability constant I II

 ML  The contribution of I and II to the real structure


M  L  ML 1  depends upon the extent of   back bonding.
 M  L  Both  donation and  - back bonding are
 ML2  synergitic.The filled orbitals CO overlap with
M  2 L  ML2 2  2 vacant metal d orbitals, which is called  dona-
 M  L  tion. The filled metal d orbitals overlap with 
antibonding moleculat orbital of CO, which is
3 
 ML3  called   back bonding.
M  3L  ML3  M  L 
3
81. Which one of the following has the highest
C - O bond length ?
n 
 MLn 2
M  nL  MLn n (A) Cr  CO 6  (B) Ti  CO 6 
 M  L 

 P ,  2 ,  3 ,  n are called over all formation con- (C)  Mn  CO 5  (D) Fe  CO5 
stants.
132 NISHITH Multimedia India (Pvt.) Ltd.,
JEE
JEEADVANCED
MAINS - CW - VOL
- VOL- VII
-I d-BLOCK & CO-ORDINATION COMPONDS
RELATIONS
82. In which one of the following M - C bond 86. Which one of the following has the highest
length is the longest ? magnetic moment ?
(A) [CoF6]3- (B) [Ni(H2O)6]2+
(A) Ti  C2 H 5 4 (B) Ni  CO 4
(C) [Fe(CN)6]3- (D) [Cu(NH3)4]2+
4
(C) Cr  CO 6 (D)  Fe  CN 6 
MATRIX MATCHING QUESTIONS
83. The C - O bond order is maximum in
2 
(A) Ti  CO 6  (B) V  CO 6  87. Match the following:
Column - I Column - II
(C)  Mn  CO 5 

(D) Cr  CO 6 Metals Properties
(A) Hf  Zr (p) density
Passage-IV
According to crystal field theory d-orbitals split (B) Au  Ag (q) IE
up in octahedral field into two sets. dxydyx dzx (C) Fe  Mn (r) MP
(D) Cu  K (s) Higher nuclear
have lower energy and d x 2  y2 and d z2 have
charge
higher energy. The difference in energy of these (t) More positive
two sets of d-orbitals is called crystal field splitting 0
ESRP value.
energy denoted by  0 .
88. Match the following:
In tetrahedral field d x 2  y2 and d z2 have lower Column - I Column - II
NISHITH Multimedia India (Pvt.) Ltd.,

energy whereas dxydyzdzx have higher energy. The Compound Characteristic


difference in energy is denoted  t . (A) TiCl4 (p) Liquid at room
temperature
0 can be determined by measuring max for
absorption and converting into energy units. (B) CaCl2 (q) Has the highest MP among
the compounds given
 0 depends upon nature of metal ions as well as
nature of ligands . The magnitude of  0 also (C) FeCl3 (r) Aqueous solution
decides low energy levels are filled. is acidic
84. Which of the following is low spin due to (D) ZnCl2 (s) Undergoes hydrolysis
strong field ligands? (t) Solid at room
(A) d 2xy d 2yz d1zx
1 1 1 1 1
B) d xy d yz d zx d x 2  y2 d z2 temperature
89 Match the following
2 1 1 1 2 1 1 2
(C) d xy d yz d zx d x 2  y 2 (D) d xy d yz d zx d x 2  y 2 Column - I Column - II
[Pair of complex [Property which compounds]
85. Given the following data about absorption is different in given(Pair]
maxima of several complex ions, the correct
order of  0 for these ions is A)  N i  C O 4  & p) Magnetic moment

Complexes max K 2  Ni  CN 4 
[Cr(H2O)6]3+ 694 nm
B) Cu NH3 4 SO4 & q) Oxidation no of central
[Cr(NH3)6]3+ 465 nm
[CrCl6]3- 758 nm K 3  Cu  CN 4  metal
3 3 3
(A)   Cr  NH 3 6    0  Cr  H 2 O 6   0 CrCl6 
0 C) K 2  NiCl4  & r) Geometry
3 3 3
B)   Cr  NH 3 6    0  Cr  H 2 O 6    0  CrCl6 
0 K 2  Ni  CN 4 
3 3 3
C)  Cr  NH 3  6    0 Cr  H 2 O 6    0  CrCl6 
0 D) K 2 [NiCl 4 ] & s) EAN of central atom
3 3 3
D) Cr  H 2O  6    0  CrCl6    0 Cr(NH 3 ) 6  K 2 [ P tC l 4 ]
0

NISHITH Multimedia India (Pvt.) Ltd., 133


d-BLOCK & CO-ORDINATION COMPONDS JEE ADVANCED - VOL - VII
90. Match the following 94. Match the following:
Column - I Column - II List-I List-II
A) MnCl24  p) Square planar A) deoxyhaemoglobin (p) Oxidation state
of the metal is +2
B) NiCl24  q) Tetrahedral
(B) Wilkinson catalyst (q) Oxidation state of
C) Cs2  CuCl4  r) Diamagnetic the metal is +1
2 (C) Sodium nitroprusside (r) Oxidation state of
D) Ni  CN 4 s) Paramagnetic
91 Match the following the metal is +3
Column - I Column - II (D) Reineck salt (s) Coordination
4 number : 4
a)  Fe  CN 6  p) Paramagnetic
(t) Coordination
2
b)  Fe  H 2O 6  q) Diamagnetic number : 6
95. Match the following
3
c)  Co  NH3 6  r) Inner orbital Complex I
3
complex (1) Co  NH 3 6  (A) 5.87 BM
NISHITH Multimedia India (Pvt.) Ltd.,

3
d)  CoF6  s) Outer orbital 3
(2)  Fe  CN 6  (B) diamagnetic
complex
t) octahedral (3)  Mn  H 2 O 6 
2
(C) 1.73 BM
92. Match the following:
Complex Characteristic II III
(A) Pt  gly2  (p) exhibits 1) (P) sp 3d 2 (U) Outer orbital complex
geometric isomerism 2) (Q) d 2 sp3 (V) Inner orbital complex
(B) Co  gly 3  (q) exhibits optical 3) (R) dsp 3 (W) Having only 
isomerism donation from the ligand
(C)  Pt  en  
2
(r) dsp 2 Choose the correct choice
 2 (A) 1: B,Q,V,W 2: C,Q, V 3: A,P,U,W
3
(D)  Co  en 3  (s) d 2 sp3 (B) 1: A,Q,V,W 2: B,P,V,W 3: B,Q,U,W
(C) 1: B,Q,U,W 2: A,P,U,W 3: C,R,V
(t) 3.87 BM (D) 1: A,P,U 2: C,R,U,W 3: A,Q,V
93. Match the following: 96. Match the following:
List-I List-II Complex I
(A) Be  acac 2 (p) Octahedral (1)  Pt  CN  4 
2
(A) 2.83 BM
2
(B)  Zn  EDTA  (q) Square planar (2)  NiCl4 2 (B) 5.87 BM
(C) K 3  Al  C2 O4 3  (r) Tetrahedral 3)  MnBr4 2 (C) 3.87 BM
(D) Ni  DMG  2 (s) Diamagnetic
4) CoCl4 2 (D) Diamagnetic
(t) Exhibits optical activity
134 NISHITH Multimedia India (Pvt.) Ltd.,
JEE
JEEADVANCED - VOL
MAINS - CW - VOL- VII
-I d-BLOCK & CO-ORDINATION COMPONDS
RELATIONS
II III 99. Statement-1:All octahedral Ni 2  complexes are
1) (P) sp 3 (U) d 8 (Low spin) paramagnetic and outer orbital complexes
whether strong or weak field ligand is present.
2) Q) dsp 2 (V) d 5
Statement-2: In octahedral Ni 2  complexes,
3) (R) Tetrahedral (W) d 7
the strong field ligands give, inner orbital
(4) (S) Square planar (X) d 8 (high spin) complexes and weak field ligands give outer
Choose the correct choice orbital complexes.
(A)1 : A,P,R,W 2 : D,Q,S,U 3: D,Q,S,W 4: A,Q,S,X
2
(B) 1 : C,Q,S,W 2 : B,P,R,W 3: A,P,R,V 4: B,Q,S,W 100. Statement-1:  NiCl4  is square planar and
(C) 1: B,P,R,V 2 : C,Q,S,V 3: C,Q,S,X 4: D,P,R,V
2
(D) 1: D,Q,S,U 2 : A,P,R,X 3: B,P,R,V 4: C,P,R,W paramagnetic whereas  PtCl4  is tetrahedral
97. Match the following and diamagnetic.
List-I List-II
2
(A) Co  en 2 Cl2  Cl (p) Geometrical Statement-2:  NiCl4  is tetrahedral and

Isomerism 2
paramagnetic but  PtCl4  is square planar and
(B) Co  en 3  Cl3 (q)Optical isomerism diamagnetic.
NISHITH Multimedia India (Pvt.) Ltd.,

(C) Cr  NH 3 4 Cl2  Cl (r) Paramagnetic 101. Statement-1:  FeF6 


3
is more stable than

(D) K 3 Cr  C2O4 3  (s) Diamagnetic  FeI 6 3


(t) Cationic complex
98. Match the following: Statement-2: Fe3 cannot oxidize F  but
Column - I Column - II Fe3 can oxidize I  to I 2 .
A)  Cr  gly 3  p) Low spin complex 2
102. Statement-1: Co  NH 3 6  is easily oxidized
3
B) Co  Br 2  Cl 2  SCN2  q) High spin complex
3
3 to Co  NH 3 6 
C) Co NH3 6  r) Optical isomerism
Statement-2: By loosing one electron from
D) Na Pt Br  ClNO2 NH3  s) Geometrical
2 3
isomerism Co  NH 3  6  , Co  NH 3 6  obeys 18e 
t) Linkage isomerism
rule.
STATEMENT TYPE QUESTIONS 103. Statement-1: Ca 2  and Mg 2  in water can be
determined by EDTA titration.
(A)Statement – 1 is True, Statement – 2 is
True; Statement – 2 is a correct explanation Statement-2: EDTA precipitates Ca 2  and
for Statement – 1. Mg 2  .
(B)Statement –1 is True,Statement – 2 is
True; Statement – 2 is NOTa correct 104. Statement-1 : Square planar complexes are
8
explanation for Statement- 1. formed by d ions with strong field ligands.
(C)Statement – 1 is True, Statement – 2 is because
False. Statement-2: The crystal field splitting is larger
(D)Statement – 1 is False, Statement – 2 is for the square planar complexes.
True.
NISHITH Multimedia India (Pvt.) Ltd., 135
d-BLOCK & CO-ORDINATION COMPONDS JEE ADVANCED - VOL - VII

115. The number of isomers possible for


INTEGER TYPE QUESTIONS
 Ir  Cl  Br  NH 3  py   is
105. In how many of the following, the second el-
ement has a higher density than the first one 116. Cr  C6 H6  2 is named as bis (  x - benzene)
? chromium ( y ) x  y is
(1) Ag , Au (2) Hf , Zr (3) Zn, Hg 117. The oxidation number of the metal in Zeises
(4) Na, Cu (5) Ca, Co (6) Ta, Nb salt is
106. The number of transition metal in bronze is 118. The oxidation state of the metal in Reinecks
x , in brass is y and inGerman silver is z . salt is  x, x is
119. The coordination number of the metal in the
Then  x  y  z  is
Vaskas complex is
107. The number of t2g electrons in 120. The oxidation number of the metal in the
3
Wilkinson catalyst is
 Fe  H 2O 6  is
121. In the complex Co  NH 3 6  Cr  CN 6 
y
108. In the complex  Al  EDTA x  ,  x  y  is x
charges over Co  NH 3 6  and
NISHITH Multimedia India (Pvt.) Ltd.,

3
109. How many of the following are sp hybrid- y
ized ? Cr  CN 6  respectively. Then x  y is
2 122. In the complex
 ZnCl4  , Ni  DMG 2 ,
2  Pt  NH 3 3 Cl  CuCl3  NH 3   , the charges
 Ni  CN 4  ,  Ni  CO 4 
x

2 2 2 1
over  Pt  NH 3 3 Cl  and
 NiCl4  ,  PtCl4  ,  Pd  NH 3  4  ,  FeCl4 
y
2 3 CuCl3  NH 3   then  x  y  is
110. How many of the following have d sp hy-
bridization ? 123. The number of stereoisomers possible in
2 3 4 Ma2 b2 c2 are
 Ni  NH 3 6  , Co  NH 3 6  ,  Fe  CN  6 
124. The number of nitrogen atoms coordinated
3 3 2
, Cr  NH 3 6  Co  en 3  , Co  EDTA   in  Zn  EDTA   is
2

3 2 125. The number of five membered rings in


CoF6  , Co  H 2O  6 
2
111. The coordination number of the metal in  Pt  trien   (trien = triethylenetetra
sodium nitroprusside is amine) is
112. The number of unpaired electrons in the
126. The oxidation state of metal is   x  , number
brown ring complex is
113. The number of five membered chelate rings of nitrogen atoms coordinated to iron  y  .
2
in  Zn  EDTA  is x  y in deoxy haemoglobin is
114. DMG is x - valent and y  dentate ligand. 127. The oxidation state and coordination number
of the metal in Wilkinson catalyst are  x
Taking x and y as numbers, find  x  y 
and y respectively.  x  y is
136 NISHITH Multimedia India (Pvt.) Ltd.,
JEE
JEEADVANCED - VOL
MAINS - CW - VOL- VII
-I d-BLOCK & CO-ORDINATION COMPONDS
RELATIONS
128. The oxidation state of the metal atom in 138. In complex [Cr(NH3)6 [Co(C2O4)3] the total
sodium nitroprusside is  x , x is No.of ions formed are
129. The number of intramolecular hydrogen 139. In complex [Fe(CO)2 (NO)2] the oxidation
bondings in Ni  DMG  2 is state of Fe is   /  
130. What are the possible number of geometri- 140. In the coordination compound K4[Ni(CN)4],
cal isomers exhibit by[CrCl2(NO2)2(NH3)2]– the oxidation state of nickel is :
? 141. The no. of statement(s) is /are
131. How many possible coordination isomers are correct ?
observed in following compound ( excluding
a) Primary valency of the central metal
the given compound)
of a complex is always satisfied by
[Cr (en)3] [Co (NO2)6]
132 A metal complex of coordination number six anions.
having three different types of ligands b) Secondary valency of the central
a, b & c of composition Ma2b2c2 can exist in metal of a complex may be satisfied by
several stereo isomeric forms. What is the either negative ions or neutral mol-
total number of such isomer? ecules.
133. The crystal field stabilsation energy c) Species which show primary valen-
cies in complex compound and are
NISHITH Multimedia India (Pvt.) Ltd.,

2
of  Mn  H 2O  6  is
present out side, can be precipitated
134. If  0 < P then electronic arrangement of out.
metal atom/ion in an octahedral complex d) Species which show secondary valen-
with d4 configuration is t 2gx egy . What is the cies in a complex are Lewis acids
142. Consider the following complex :
value of x
135. The no. of complexes showing fac and mer [Co(NH3)5CO3]ClO4
isomerism the coordination number is X and oxi-
3 dation number is Y , number of d– elec-
A) Co  NH 3 3 Cl3  B) Co  en3   trons is Z, and number of unpaired d–
electrons of the metal is A are respec-
C) Co  gly 3 
tively.  X  Y    Z  A  .
D) Co  NH 3 2  H 2O 2 Cl2  4
143. The CFSE for  CoCl6  is 18000cm–1. The
2 2
E)  Fe  dipy 3   for  CoCl4  will be Y  1000 cm 1 the

2
value of y is
F)  Fe  O  phen 3  3 3
144. Amongst  Co  OX 3  , Co F6  ,
136. The no. of complexes showing hepticity 5
3
1) ferrocene 2) zeise’s salt  C o  N H 3 6  and
3) ruthenocene 4) chromocene
2
137. The no. of flexible ligands are Co  NH 3 6  the no. of coloured
 
a) Sulphato b) Edta complexes
c) nitrato d) thiosulphato
NISHITH Multimedia India (Pvt.) Ltd., 137
d-BLOCK & CO-ORDINATION COMPONDS JEE ADVANCED - VOL - VII
145. This spin magnetic moment of cobalt in
MATRIX MATCHING TYPE
Hg Co  SCN 4  is x. The no. of 87) A: p, q, r, s, t ; B: p,q,r,s,t
C: p,q,r,s,t ; D: p,q,r,s,t
unpaired electrons are 88)A: p,r,s ; B: q,t ; C : r,s,t ; D: r,s,t
146. From Ma3b3 , when 1b is replace by c, the 89)A-q,r,s B- p,q,r,s; C-p, r D- p,r,s
90)A-q,s B- q,s C-q,s D- p,r
total no. of geometrical possible are
91) A-q,r,t B- p,s,t ; C-q,r,t ; D- p,s,t
92)A: p,r ; B: p,q,s ; C: r ; D: q,s
93)A - q,s,t B - p,s,t C- p,s,t ; D- q,s
LEVEL-VI 94) A - p,t ; B - q,s; C - p,t ; D - r,t
95) A 96) D
KEY 97) A - p, q, s,t B - q,s,t C-p, r, t D-q, r
SINGLE ANSWER 98) A - r,s B - q,r,s,t C - p D - s,t
1) B 2) C 3) C 4) A 5) A
6) A 7) C 8) A 9 )A 10) C ASSERTION AND REASON TYPE
11)D 12) C 13) D 14) C 15) D 99) C 100) D 101) A
16) D 17) B 18) C 19) C 20) B 102) A 103) C 104) A
21)D 22) D 23) A 24) B 25) D
NISHITH Multimedia India (Pvt.) Ltd.,

26) C 27) C 28) A 29) B 30) B INTEGER TYPE


31) C 32) B 33) B 34) D 35) C 105) 4 106)4 107)3 108) 2 109) 4
36) C 37) A 38) D 39) D 40) B 110) 5 111) 6 112) 3 113) 5 114) 3
41) B 42) C 43) C 44) B 45) B 115)3 116) 6 117) 2 118) 3 119) 4
46) B 47) A 48) D 49) D 50) C 120)1 121) 6 122) 2 123)6 124) 2
51) C 125)3 126)7 127)5 128) 2 129) 2
MULTIPLE ANSWER 130) 5 131) 3 132) 6 133) 0 134) 3
52) A,C 53) A,B 54) A,B 135) 2 136) 2 137) 3 138) 4 139) 2
55) A,B,D 56)A,B,D 57) A,B,C 140) 0 141) 3 142) 3 143) 8 144) 4
58)A,B,C,D 59)A,B,D 60) A,B,D 145) 3 146) 3
61) A,B,C,D 62)A,B,D 63)B,C,D
64)A,B,D 65) A,B,C,D 66) A,B,C,D LEVEL - VI
67) A,C,D 68) A,B,C,D 69) A,B,C HINTS
70) A,C 71) A,D 72) A,B,C,D
73) B,C 74) A,C
SINGLE ANSWER QUESTION
COMPREHENSION TYPE
1. Au has higher electron affinity and so can form
Passage-I
75) C 76) C 77) A Au  readily..
Passage-II 2 Pd and Pt in VIII B group. IE4  IE4  Ni 
78) A 79) C 80) B
Passage-III 4. Famous alloy for permanent magnets
81) B 82) A 83) C 5. Ti3 3d 1 ,V 2 ,3d 3 . Others have d 0 or d 10 con-
Passage-IV
84) A 85) A 86) A figuration. CuSO4 has no splitting of d orbitals.
No d  d transition is possible
chelate .
138 NISHITH Multimedia India (Pvt.) Ltd.,
JEE
JEEADVANCED - VOL
MAINS - CW - VOL- VII
-I d-BLOCK & CO-ORDINATION COMPONDS
RELATIONS
71. A is tetrahedral, D is unsymmetrical, B and C
8. V  CO 6  is paramagnetic with 1 unpaired are square planar and not optically active.
electron, all others are diamagnetic. 72. Apply Pauling VB theory. See the structures.
2 2 4
39. NO, NO  , NO  can act as ligands. 73. t    0
3 3 9
40. V 4  : 3d 1 , Cu 2 , 3d 9 . So they have same color.. 74. (A) and (C) are correct about Wilkinson’s cata-
41. 2 x  8  0  0  2 , x  3 lyst whereas (D) is Ziegler-Natta catalyst.
42. See the structure Wilkinson’s catalyst is  Ph3 P 3 RhCl 
43. 3.87 BM, it has 3 unpaired electrons.
44. The more negative charge will give more elec- COMPREHENSION TYPE
trons into  * MO of CO. So the bond order
decreases. Passage-I
45. Use Pauling VB theory.
75,76,77 x : Cr  NH 3 6  Co  CN 6  ;
46. (II) has symmetry.
47. It is a  complex. y : Co  NH 3 6  Cr  CN 6 
48. In other complexes the orientation decides
whether the complex is optically active or not.
Passage-II
49. Coordination isomerism
NISHITH Multimedia India (Pvt.) Ltd.,

50. In A all the coordinating atoms are identical, no 78. K1K 2 K 3   3 ,  n  K1K 2 .....K n ,
geometrical isomerism. log  n  log K1  log K 2  log K3  .....  log K n
51. Note that 2 NO  3CO . NO is 3e  donor and 79.
2 2
Cd  H2O4   NH3  Cd  H2O3  NH3   H2O
CO is 2e  donor.. Any of the four water molecules can be replaced
by one NH 3 molecule. Statistical or probability
MULTIPLE ANSWER QUESTIONS factor favours.
80. Both the reations involve Cd - N bonds and so
53. Have oxidation number greater than the group no much change in enthalpy. In (1) four water
number. molecules are replaced by four NH 3 molecules.
54. V 5 , Zn 2 react with both acids and basis, are But in (2) four water molecules are replaced by
amphoteric. two en molecules. This increases the entropy and
55. Do not contain Iron. is called as Entropy effect.
65. Consider outer electronic configuration. G 0   H 0  T  S 0
E g :  G 0   2.303 RT log 
Cr  CO 6  : Cr  O  : d 4 s 2 , 6  6  2  18e  In (2), G0 varies due to S 0 and so the log  2
has a higher value. For (1), S 0   ve and (2)
C6 H 6 and C5 H 5  are 6 e systems.
S 0   ve
66. All obey EAN rule.
67. B is a 6 coordinated complex. Passage-III
68 Ni 2  : 3d 8 , All octahedral Ni 2  complexes are 81 More negative charge on Ti atom, which pumps
3 2
sp d and paramagnetic. more electrons into  MO of CO, So Bond
order decreases, bond length increases.
69. RMgX is a  complex. No  - bond. 82. Metal - alkyl. No  - back donation. Only 
70. B and D have symmetry donor.
NISHITH Multimedia India (Pvt.) Ltd., 139
d-BLOCK & CO-ORDINATION COMPONDS JEE ADVANCED - VOL - VII
83. Note positive charge on the complex. Due to
less electron density over the metal, less elec- INTEGER TYPE QUESTIONS
trons are given into the  MO of CO.
105. Three row transition elements have higher den-
Passage-IV sity than second row elements. d-block elements
84. Electrons are filled in t2g orbitals have higher density than group IA and IIA ele-
ments. Note that atomic size decreases or re-
hc mains almost same and atomic weight increases
85. 0 
 and so show higher density.
86. Cobalt (III), d 6 , Highspin complex, 4 unpaired 106. Bronze:
electrons Cu,Sn; Brass : Cu, Zn; German silver : Cu, Zn, Ni
11 2  4
MATRIX MATCHING TYPE 3 2
107. t  e 
2g g

87. II and III row transition elements have similar 1


atomic radii but with increase in atomic weight, 108.  Al  EDTA 
so density is more for III row elements.
109.  ZnCl4 2 , Ni CO 4 ,  NiCl4 2 ,  FeCl4 1
94 Reineck salt NH 4 Cr  NH 3 2  NCS 4 
NISHITH Multimedia India (Pvt.) Ltd.,

are tetrahedral and sp 3 hybridized.


It is a dark red crystalline compound
It is used to precipitate primary and secondary 110. They are
amines as their ammonium salts 3 4
Co  NH 3 6  ,  Fe  CN  6  ,
3 3 2
ASSERTION AND REASON TYPE Cr  NH 3  6  , Co  en 3  , Co  EDTA  

99. Ni 2 : 3d 8       111. Na2  Fe  CN 5  NO  


Note that by pairing up d 2 sp 3 cannot be 112. X X X X X X
obtained in the case of strong field ligands.
100.  0 Pt 2    0 Ni 2  , The Cl  ligand could pair 113. See the structure of the complex.
up e in 5d orbitals but could not pair up e in 114. univalent bidentate ligand.
3d orbitals. 115. Square planar complex
101. Fe3 is hard acid and F  is a hard base (Pearson bis  6  benzene  chromium
concept). 116.
O , 6  0  6
Stable complex is formed. Further Fe3 can
oxidize I  to I 2 . 117. K  PtCl3  C2 H 4  
2
102. Co  NH 3 6  , has 7  6  2  19e  118.  NH 4  Cr  NCS 4  NH 3 2 

Co  NH 3 6 
3
has 6  6  2  18e  119.  Ir  Cl  CO  PPh3  2 

18e  is more stable. 120.  RhCl  PPh3 3 


103. EDTA forms complex with Ca 2  and Mg 2  . 3 3
121. Co  NH 3 6  and Cr  CN 6 

140 NISHITH Multimedia India (Pvt.) Ltd.,


JEE
JEEADVANCED - VOL
MAINS - CW - VOL- VII
-I d-BLOCK & CO-ORDINATION COMPONDS
RELATIONS
1 1 141. Werner’s theory
122.  Pt  NH 3 3 Cl  and CuCl3  NH 3  
142. C.N. 6, O.S. 3, No. of d-electrons 6
11  2
123. Give both geometrical and optical isomers (3 + Co 3  3d 6  , unpaired electrons 0.
 
1) .
124. See the structure of the complex. 4
143. t  0
125. NH 2CH 2CH 2 NHCH 2CH 2 NHCH 2CH 2 NH 2 9
126. Iron is in +2, 4 pyrrole nitrogens and one histidine 144. d 1 to d 9 coloured
nitrogen 2 + 4 + 1 = 7. 145. Co is in +2 O.S. 3d 7 configuration.
127.  RhCl  PPh3 3  , 1  4  5

Na2  Fe  CN 5  NO   ,  a
128. a
2  x  5  1  0, x  2 b
129. See the structure.
3 3 M
131. 1) Cr  en 3  , Co  NO2 6  146.
NISHITH Multimedia India (Pvt.) Ltd.,

  a
2) Cr  en 2  NO2 2  , Co en  NO2 4  c
b
 
3) Co  en 2  NO2 2  , Cr en  NO2 4 

4) Co  en 3  , Cr  NO2 6  a


a
133. CFSE   0.6 p  0.4q   0 , t23g eg2 , CFSE  0 b
p and q are the number of electrons in t2g and
M
eg orbitals.
b
134. t  e 
3
2g
1
g c
n a
135.  Ma3b3 n and  M  AB 3  show fac and
mer isomerism.
136. Ferrocene and osmocene are linked to a
cyclo pentadienylo ion, which contains 5 carbon a
atoms in aring. . c
137. Conceptual
M
138.  Cr(NH3 )6 3 is 1 ion and
[Co(C2O 4 )3 ]3 is other b
b
139. NO is a positive ligand
140. Cyanide is a negative ligand a

NISHITH Multimedia India (Pvt.) Ltd., 141


d-BLOCK & CO-ORDINATION COMPONDS JEE ADVANCED - VOL - VII
5 The correct structure of
PREVIOUS IIT QUESTIONS ethylenediaminetetraacetic acid (EDTA) is
(IIT2010)
1  
NiCl 2 P  C 2 H 5  2  C 6 H 5  exhibits HOOC  CH 2 CH 2  COOH

temprature dependent magnetic bahaviour N CH N


(para magnetic and dia magnatic) .The co- (A)
HOOC  CH 2 CH 2  COOH
ordination geomatries of Ni 2 in the
paramagnetic and diamagnetic states are HOOC COOH
respectively (IIT2012 ) N CH 2 CH 2 N
(B)
A) Tetrahedral and tetrahedral HOOC COOH

B) Square planar and square planar HOOC  CH2 CH2  COOH


C) tetrahedral and square planar
N CH 2 CH2 N
D) Square planar and tetrahedral (C)
HOOC  CH2 CH2  COOH
2 As per IUPAC nomenclature, the name of
COOH
the complex Co  H 2O 4  NH3 2  Cl2
HO O C  C H 2 CH 2
H
(IIT-2012)
NISHITH Multimedia India (Pvt.) Ltd.,

N CH CH N
A) Tetra aqua diamine cobalt (III) chloride (D) H
CH2 CH 2  CO O H
B) Tetra aqua diammine cobalt (III) chloride HOOC
C) Diamine tetra aqua cobalt (III) chloride
6 The ionization isomer of (IIT2010)
D) Diammine tetraaqua cobalt (III) chloride
3 Geometrical shapes of the complexes  Cr  H 2 O 4 Cl  NO 2   Cl is

formed by the reaction of Ni 2 with (A)  Cr  H 2O 4  O 2 N   Cl 2


Cl,CN andHO
2
, respectively, are (IIT2011) (B)  C r  H 2 O  4 C l 2   N O 2 
A) Octahedral , tetrahedral and square planar
(C)  Cr  H 2 O 4 Cl  ONO   Cl
B) tetrahedral, square planar and octahedral
C) square planar, tetrahedral and octahedral (D)  Cr  H 2O 4 Cl 2  NO 2   H 2 O
D) octahedral, square planar and octahedral 7 Total number of geometrical isomers for the
4 Among the following complexes  K  P  complex
 RhCl  CO  PH 3  NH 3   is (IIT2010)
K 3  Fe  CN  6  ,  Co  NH 3 6  Cl 3  L  ,
8 The complex showing a spin- only magnetic
Na 3  Co  oxalate 3   M  , moment of 2.82 B.M is (IIT2010)
(A) Ni  CO 4 (B) [NiCl4 ]2 
 Ni  H 2 O 6   Cl 2  N  ,
2
(C) Ni  PPh 3 4 (D)  Ni  CN 4 
and  Zn  H 2 O  6   NO 3  2  P 
9 The spin only magnetic moment value (in
the diamagnetic complexes are
Bohr magneton units )of Cr  CO 6 is
A) K, L, M, N B) K, M, O, P
(IIT-2009)
C) L, M, O, P D) L, M, N, O
(A) 0 (B) 2.84 (C) 4.90 (D) 5.92

142 NISHITH Multimedia India (Pvt.) Ltd.,


JEE
JEEADVANCED - VOL
MAINS - CW - VOL- VII
-I d-BLOCK & CO-ORDINATION COMPONDS
RELATIONS
10 The compound (s) that exhibit(s) geometri- 17. The pair(s) of coordination complexes/ions
cal isomerism is (are) (IIT-2009) exhibiting the same kind of isomerism is
( are )
(A)  Pt  en  Cl2  (B)  Pt  en  2  Cl2
A)  CrNH 3 5Cl  Cl2 and  CrNH 3 4Cl2  Cl
(C)  Pt  en 2 Cl 2  Cl 2 (D) Pt  NH3 2 Cl2   
11 Among the following, the coloured compound B)  Co NH 3 4Cl2  and  Pt NH 3 2 H 2O Cl 
is [IIT -2008] C) CoBr2Cl2 2 and  PtBr2Cl2 2
(A) CuCl (B) K3[Cu(CN)4]
(C) CuF2 (D) [Cu(CH3CN)4]BF4 D)  Pt  NH3  3 NO3  Cl and  Pt  NH3  3Cl  Br
12. Both [Ni(CO)4] and [Ni(CN)4]2- are diamag-
netic. The hybridisations of nickel in these
complexes, respectively, are [IIT-2008] PREVIOUS IIT KEY
(A) sp3, sp3 (B) sp3, dsp2 1) C 2) D 3) B 4) C
3
(C) dsp ,sp 3
(D) dsp2, dsp2 5) C 6) B 7) 3 8) B
13 The coordination number of Ni2+ is 4 9) A 10) C,D 11) C 12) B
NiCl2 + KCN (excess)  A(Cyano 13)A 14) C 15) A 16) B
complex) 17) B, D
NiCl2 + Conc. HCl(excess)  B (chloro com-
plex) SOLUTIONS
NISHITH Multimedia India (Pvt.) Ltd.,

. The IUPAC name of A and B are


(A) Potassium tetracyanonickelate(II), Potassium
tetrachloronickelate(II) 1. Ni 2   1s 2 2s 2 sp 6 3s 2 3p6 4s 2 3d 8
(B)Tet racyano po t assiumnickelat e(II), low spin complex : dsp 2  square planar
teterachlorpotassiumnickelate(II) ( diamagnetic )
(C) Tetracyanornickel (II), tetrachloronickel(II)
(D) Potassium tetracyanonickel (II), Potassium high spin complex : sp3  tetrahedral
terachloronickel(II) [IIT-2006] ( paramagnetic )
14. Predict the magnetic nature of A and B 2.  Co  H 2 O  4  NH 3  2  Cl3
(A) both are diamagnetic
(B) A is diamagnetic and B is paramagnetic with According IUPAC nomenclature rule it is
one unpaired electron Diamminetetraaquacobalt (III) chloride i.e.,
(C) A is diamagnetic and B is paramagnetic with NH3 is neutral ligand and it is named as ammine
two unpaired electrns.
and H 2 O  aqua 
(D) Both are paramagnetic [IIT-2006]
15. The hybridization of A and B are [IIT-2006] 3. Cl is weak field ligand and CN  is a strong
(A) dsp2, sp3 (B) sp3, sp3 field ligand, Ni 2 with chloride and Cyanide has
(C) dsp2, dsp2 (D) sp3d2 , sp3 C.N. 4.
16. Consider the following complex ions, P, Q With water has C.N. 6.
3
and R. P   FeF6  , Q   V  H 2 O 6 
2
and 4. Complexes L, M, O form inner orbital com-
plexes due to high charge density metal, or high
2 Q. N of metal.
R   Fe  H 2 O  6  . The correct order of the
Zn 2 with C.N. 6 3d configuration has no
10
complex ions, according to their spin - only unpaired electrons.
magnetic moment values ( in BM ) is
[ IIT - 2013 ] 7.  RhCl  CO  PH 3  NH 3   is an example of
A) R  Q  P B) Q  R  P
 Mabcd n  type, squareplanar geometry hav-
C) R  P  Q D) Q  P  R ing three geometrical isomers.
NISHITH Multimedia India (Pvt.) Ltd., 143
d-BLOCK & CO-ORDINATION COMPONDS JEE ADVANCED - VOL - VII
8. Complex Ni  CO  4 and Ni  PPh 3  4 are tetra-    25BM
hedral complexes with Ni in 0 O.S., complex PR Q
2
 Ni  CN  4  is a squareplanar, and all these 17. A)  CrNH 3 5Cl  Cl2 does not exhibit isomeris,
three are diamagnetic. CrNH 3 Cl2  Cl exhibits geometrical
4
2
complex  NiCl4  is tetrahedral complex with iosmerism

Ni in +2 O.S., paramagnetic with 2 unpaired B)  Co NH 3 4Cl2  and
electrons.

9. Cr  CO 6 is octahedral geometry with Cr in 0  Pt NH 3 2 H 2O Cl  both can show

O.S. geometcial isomerism


C) CoBr2 Cl22 tetrahedral so it does not exhibit
10.  Pt  en  2 Cl 2  Cl 2 this complex is octahedral
isomerism
with Pt in +4 O.S . and C.N. 6.
n PtBr2 Cl22  - squareplanar so it exhibits geo-
 M  AA  2 a 2  type.
metrical isomerism
 Pt  NH 3  2 Cl2  this complex is squareplanar D)  Pt NH 3 3 NO3  Cl and  Pt NH 3 3Cl  Br
NISHITH Multimedia India (Pvt.) Ltd.,

geometry with Pt in +2 O.S. and C.N. 4. both can show inonisation isomerism
 Ma 2 b 2 n  type.
11. Cu in +2 has 3d9 configuration can have d  d
trasition, coloured
12. In complex  Ni  CO 4  Ni is in 0 O.S., tetra-
hedral.
2
In complex  Ni  CN  4  Ni is in +2 O.S.,
squareplanar.
13. Conceptual
14 & 15. Complex is K 2  Ni  CN 4  , squareplanar

complex b is K 2  Ni  Cl  4  tetrahedral.

16.  FeF6 3 , Fe3 has 5 unpaired electron

   35BM
2
  V  H 2 O 6  , V 2 has 3 unpaired elec-
trons
   15BM
2
  Fe  H 2 O  6  , Fe 2  has 4 unpaired elec-
trons
144 NISHITH Multimedia India (Pvt.) Ltd.,

You might also like